Pulmonary Part 1. (1-100)

Pataasin ang iyong marka sa homework at exams ngayon gamit ang Quizwiz!

7. A 24-year-old man is admitted to the critical care unit after sustaining a pulmonary contusion in a motor vehicle collision. He has no history of cardiac or pulmonary disease. Over the last few hours he has been complaining of increasing dyspnea, his respiratory rate has been increasing, and his oxygen saturation via pulse oximetry has been decreasing. Breath sound assessment reveals fine crackles bilaterally. Arterial blood gases reveal respiratory alkalosis and hypoxemia. Chest x-ray film reveals patchy infiltrates. Acute respiratory distress syndrome is diagnosed. Oxygen therapy is initiated, and arterial blood gases are monitored closely. Because the patient is becoming fatigued and arterial blood gases now reveal respiratory acidosis and severe hypoxemia, even on 100% oxygen, it is decided that he must be intubated and mechanically ventilated, and positive end-expiratory pressure (PEEP) must be instituted slowly. What is the rationale for using PEEP in this patient? a. To increase the driving pressure of oxygen b. To treat metabolic acidosis c. To enhance carbon dioxide elimination d. To allow better removal of secretions

Correct answer: a Rationale: PEEP has three primary purposes: increasing the driving pressure of oxygen, decreasing surface tension and the work of breathing, and decreasing shunt by reopening collapsed alveoli. Driving pressure of oxygen is determined by multiplying the fraction of inspired oxygen (FiO2) and barometric pressure. Increasing the pressure can decrease the FiO2 while maintaining the same PaO2 or a better PaO2 on the same FiO2. PEEP is not used to treat metabolic acidosis, enhance carbon dioxide elimination, or allow better removal of secretions. Test-Taking Strategy: Link driving pressure with positive end-expiratory pressure. <System> Pulmonary <Section> Mechanical Ventilation <Level> Synthesis/Evaluation

83. If a patient with a nasal cannula at 2 L/min breathes more slowly or more shallowly, what will happen to the inspired oxygen concentration?a. Fraction of inspired oxygen (FiO2) will increase.b. FiO2 will decrease. c. FiO2 will not change.d. FiO2 will be less effective in oxygenating the blood.

Correct answer: aRationale: A nasal cannula is a low-flow oxygen delivery system. The concentration of oxygen will increase or decrease depending on the amount of diluent (room air) that the patient takes in with the oxygen from the oxygen delivery system. If the patient breathes more slowly or shallowly, the amount of diluent decreases and the inspired oxygen concentration increases. If the patient breathes more rapidly or deeply, the patient still gets only 2 L/min of oxygen from the nasal cannula but more diluent (room air), resulting in a decrease in inspired oxygen concentration.Test-Taking Strategy: Remember that low-flow oxygen delivery systems do not ensure a given inspired oxygen concentration and should be avoided if respiratory rate or tidal volume are significantly increased or decreased above normal. Consider that they cannot pull more or less oxygen from the nasal cannula and the wall, so the only factor left to change is the amount of room air diluent. More diluent equals lower inspired oxygen concentration. Less diluent equals higher inspired oxygen concentration. Choose option a. <System> Pulmonary<Section> Oxygen<Level> Application/Analysis

47. A 36-year-old woman with a long history of asthma is admitted to the critical care unit from the emergency department. Blood pressure is 110/80 mm Hg, heart rate is 110 beats/min, and respiratory rate is 28 breaths/min. Arterial blood gas results on 24% oxygen are as follows: pH 7.48 PaCO2 29 mm Hg PaO2 59 mm Hg HCO3 26 mEq/L Which change would indicate that the patient's ventilation status is worsening? a. PaCO2 is now normal.b. PaO2 is now normal.c. HCO3 is now elevated above normal. d. HCO3 is now decreased below normal.

Correct answer: aRationale: A normalization or increase in the PaCO2 in asthmatic patients is an ominous sign. This patient is still tachypneic, and if ventilation is normal, PaCO2 should be decreased. The other ominous sign in this patient would be absence of wheezing or rhonchi, because they would indicate that ventilation is insufficient to cause these noises. Test-Taking Strategy: PaCO2 reflects ventilation. When you see ventilation, think PaCO2. <System> Pulmonary<Section> Asthma<Level> Synthesis/Evaluation

51. Optimal positive end-expiratory pressure (PEEP) is the amount of PEEP required to do which of the following?a. Achieve arterial oxygen saturation (SaO2) greater than 90% without significantly reducing cardiac output b. Achieve the highest SaO2c. Decrease static compliance to less than or equal to 50 mL/cm H2O d. Achieve the highest cardiac output

Correct answer: aRationale: A primary purpose of PEEP is to increase driving pressure of oxygen and improve oxygenation of the arterial blood. An adverse effect of PEEP is a decrease in venous return and cardiac output. Because oxygen delivery is affected by SaO2, hemoglobin, and cardiac output, an increase in SaO2 to the point of a decrease in cardiac output does not improve oxygen delivery. So optimal (or "best") PEEP is the PEEP that will achieve an SaO2 greater than 90% without decreasing the cardiac output. PEEP increases compliance, and it does not increase cardiac output.Test-Taking Strategy: Notice that option a includes two factors that affect oxygen delivery. Choose option a. Another approach is to eliminate options b and d because they are too general; highest is a nonspecific qualifier. If deciding between options a and c, go with what you know, option a.<System> Pulmonary<Section> Mechanical Ventilation<Level> Application/Analysis

87. Which of the following is not a likely cause of respiratory acidosis?a. Pulmonary embolismb. Overdosage of central nervous system depressant drugc. Neuromuscular disease such as Guillain Barré, polio, or myasthenia gravis d. Chronic obstructive pulmonary disease with pulmonary infection

Correct answer: aRationale: All of these are potential causes of acute respiratory failure. Acute respiratory failure with hypoxemia without hypercapnia is called type I, and hypoxemia with hypercapnia is called type II. Options a, b, and c are causes of type II acute respiratory failure, but pulmonary embolism is a cause of type I acute respiratory failure because it affects oxygenation by affecting perfusion and diffusion rather than ventilation. Test-Taking Strategy: Note that this is a negatively stated question. Consider which option does not cause hypercapnia, which would cause respiratory acidosis.<System> Pulmonary<Section>Acute Respiratory Failure; Arterial Blood Gas<Level> Application/Analysis

37. A 72-year-old woman is admitted to the critical care unit in acute respiratory distress. Her respiratory rate is 38 breaths/min and labored. Her blood pressure is 94/68 mm Hg and heart rate is 116 beats/min. The monitor shows atrial fibrillation. Lung auscultation reveals crackles in the lung bases. Ventilation-perfusion scan results indicate a high probability of pulmonary embolism. Which of the following describes the pathophysiology of pulmonary embolism? a. Increased alveolar dead space b. Increased intrapulmonary shunt c. Hypoventilationd. Decreased inspired oxygen

Correct answer: aRationale: Alveolar dead space is ventilation without perfusion (also referred to as high ventilation/perfusion [V/Q] ratio). Intrapulmonary shunt is perfusion without ventilation (also referred to as low V/Q ratio). Patients with pulmonary embolus actually hyperventilate until respiratory muscle fatigue occurs. Decreased inspired oxygen would indicate that the patient is inspiring less than 21% oxygen. This occurs with smoke inhalation.Test-Taking Strategy: To remember what a dead space unit is, think of anatomic dead space. Anatomic dead space is from the nose down to and including the terminal bronchioles. This area has air (ventilation) but no alveolar-capillary membrane (perfusion). In a pulmonary embolus, there is air in the alveolus, but no blood is available to that area to pick up the oxygen in the alveolus. Choose option a.<System> Pulmonary<Section> Pulmonary Embolism<Level> Knowledge/Comprehension

58. A patient with myasthenia gravis develops hypoxemia with hypercapnia. What is the cause of hypoxemia in this patient?a. Alveolar hypoventilationb. Alveolar hyperventilation c. Diffusion defect d. Shunt

Correct answer: aRationale: Alveolar hypoventilation causes hypoxemia and hypercapnia. Alveolar hyperventilation would cause hypocapnia. Diffusion defect or shunt would cause hypoxemia with normal or decreased PaCO2, depending on ventilation.Test-Taking Strategy: Remember that PaCO2 is an indication of ventilation and PaO2 is an indication of oxygenation. Thus an increase in PaCO2 is an indication of alveolar hypoventilation (option a), which also causes hypoxemia because the elevated partial pressure of carbon dioxide displaces oxygen in the alveolus (i.e., Dalton's law).<System> Pulmonary<Section> Acute Respiratory Failure<Level> Application/Analysis

72. A patient on a mechanical ventilator gains weight and has decreased urine output. What ventilatory parameter would provide additional evidence of fluid volume excess? a. Increased A:a gradientb. Increased vital capacity c. Increased lung complianced. Decreased peak inspiratory pressure

Correct answer: aRationale: An A:a gradient greater than 10 mm Hg is a reflection of a diffusion defect. Note that the process of diffusion is between A (alveolus) and (a) arterial blood. An increase in intraalveolar fluid dilutes and inactivates surfactant, causing alveolar collapse (decreasing vital capacity) and decreases lung compliance. Peak inspiratory pressure increases reflect a decrease in lung compliance.Test-Taking Strategy: If you do not know the answer, cluster the options. Options b, c, and d are positive or desired changes. Only option a is negative and could reflect intraalveolar fluid.<System> Pulmonary<Section> Mechanical Ventilation<Level> Synthesis/Evaluation

30. A 52-year-old man had surgery yesterday for a ruptured abdominal aortic aneurysm. He has a history of hypertension and coronary artery disease but no history of pulmonary problems. He is intubated and mechanically ventilated during the night after surgery. The next morning the patient is on intermittent mandatory ventilation (IMV) with a rate of 10 breaths/min, tidal volume of 650 mL, fraction of inspired oxygen of 0.4, and positive end-expiratory pressure (PEEP) of 5 cm H2O. Vital signs are stable, and his total respiratory rate is 16 breaths/min. Arterial blood gases are pH, 7.39; PaCO2, 42 mm Hg; HCO3, 22 mEq/L; and PaO2, 96 mm Hg. In preparation for weaning, spontaneous ventilatory parameters are measured. Spontaneous tidal volume is 400 mL, vital capacity is 950 mL, minute ventilation is 9 L/min, respiratory rate is 22 breaths/min, and maximal inspiratory pressure is −29 cm H2O. The patient weighs 69 kg. Based on this information, what form of weaning is likely to be implemented? a. Short-term breathing trial with pressure supportb. Intermittent mandatory ventilation with diminishing tidal volume over time c. Intermittent mandatory ventilation with diminishing rate over timed. Short-term breathing trial with continuous positive airway pressure (CPAP)

Correct answer: aRationale: Based on these excellent weaning parameters, a short-term breathing trial with pressure support is indicated. Remember that pressure support reduces work of breathing while CPAP increases the driving pressure of oxygen and improves oxygenation. If the patient had required more than physiologic PEEP (5 cm H2O) to maintain an acceptable PaO2 but ventilatory parameters were good, continuous positive airway pressure might have been considered. There is no indication of persistent oxygenation problems, so pressure support would be indicated more than CPAP. In addition, the short duration of mechanical ventilation makes it less likely that he has ventilatory muscle deconditioning, which would require IMV.Test-Taking Strategy: Consider that for short-term mechanical ventilation, a short-term breathing trial with pressure support is the most common strategy for weaning. <System> Pulmonary<Section> Mechanical Ventilation<Level> Synthesis/Evaluation

73. A 65-year-old woman reports severe dyspnea 2 days after abdominal surgery. She is transferred to the critical care unit. On 5 L of oxygen by nasal cannula, her arterial blood gases are as follows: pH- 7.39 PaCO2- 35 mm Hg HCO3- 19 mEq/L PaO2- 40 mm Hg Arterial oxygen saturation- 75% Why does this patient have hypoxemia without hypercapnia? a. Because carbon dioxide is more diffusible than oxygenb. Because carbon dioxide has more driving pressurec. Because carbon dioxide is less diffusible than oxygen d. Because carbon dioxide excretion by the kidney is increased

Correct answer: aRationale: Carbon dioxide is 20 times more diffusible than oxygen. In conditions that affect diffusion but do not affect ventilation, expect the PaO2 to be decreased and the PaCO2 to be normal (or decreased in hyperventilation, as in this patient). If ventilation were affected, such as if this patient were fatiguing, the PaCO2 then would increase. Driving pressure is the fraction of the gas in inspired air multiplied by the barometric pressure. Because carbon dioxide is ~0.5% of inspired air, the driving pressure would be very low. The kidney eliminates bicarbonate and hydrogen ions, but the lungs eliminate carbon dioxide. Test-Taking Strategy: Note that options a and c are direct opposites. Both are not likely be correct, and option d would result in an increase in PaCO2. Because the PaCO2 is low, choose option a.<System> Pulmonary<Section> Acute Respiratory Failure<Level> Application/Analysis

94. Which of the following is not a purpose of chest physical therapy? a. Decrease dead spaceb. Decrease airway resistancec. Decrease shunt d. Increase mucociliary clearance of secretions

Correct answer: aRationale: Chest physical therapy increases the movement of secretions, which would decrease shunt and airway resistance. Dead space occurs when there is a decrease in perfusion to a ventilated area.Test-Taking Strategy: Three of the options have to do with secretions and ventilation, and one deals with perfusion. Choose option a.<System> Pulmonary<Section> Airway<Level> Application/Analysis

44. A patient is admitted to the critical care unit with multiple trauma resulting from a motor vehicle collision. She is becoming increasingly dyspneic. Of the following serious consequences of chest trauma, which is the most life-threatening?a. Tension pneumothorax b. Closed pneumothoraxc. Subcutaneous emphysema d. Open pneumothorax

Correct answer: aRationale: During a tension pneumothorax, air enters the pleural space with each breath but cannot escape. This compresses the heart and the unaffected lung, and causes mediastinal shift. Mediastinal shift may cause compression or tearing of the great vessels if not promptly relieved. Tearing of the great vessels (e.g., aorta) is almost always fatal. Test-Taking Strategy: Tension implies pressure. Tension pneumothorax is logically more serious than the other forms of pneumothorax or subcutaneous emphysema.<System> Pulmonary<Section> Thoracic Trauma<Level> Application/Analysis

36. A 72-year-old woman is admitted to the critical care unit in acute respiratory distress. Her respiratory rate is 38 breaths/min and labored. Her blood pressure is 94/68 mm Hg and heart rate is 116 beats/min. The monitor shows atrial fibrillation. Lung auscultation reveals crackles in the lung bases. Ventilation-perfusion scan results indicate a high probability of pulmonary embolism. Which of the following is the most frequently seen clinical indicator of pulmonary embolism? a. Dyspneab. Hemoptysis c. Coughing d. Chest pain

Correct answer: aRationale: Dyspnea, tachycardia, tachypnea, and anxiety are the most frequently seen clinical indicators of a pulmonary embolus. Chest pain also may occur but is not as likely as dyspnea.Test-Taking Strategy: Think of a patient that you have seen with a pulmonary embolus. The most pronounced symptom was most likely dyspnea. Choose option a.<System> Pulmonary<Section> Pulmonary Embolism<Level> Knowledge/Comprehension

89. Which of the following acid-base imbalances would be expected early in acute respiratory distress syndrome (ARDS)?a. Respiratory alkalosisb. Respiratory acidosis c. Metabolic acidosis d. Metabolic alkalosis

Correct answer: aRationale: Early in ARDS the patient hyperventilates, and the arterial blood gases show respiratory alkalosis with hypoxemia. As pulmonary compliance is reduced, ventilation becomes more difficult, and the patient fatigues. Respiratory acidosis then occurs. Test-Taking Strategy: Rule out the metabolic conditions first because this is clearly a respiratory problem. Next, remember that ARDS is a cause of type I acute respiratory failure. The oxygen is down, but the PaCO2 is down too because the patient is hyperventilating.<System> Pulmonary<Section> Acute Respiratory Distress Syndrome; Arterial Blood Gas<Level> Knowledge/Comprehension

45. A 70-kg patient has been on a mechanical ventilator after surgery. He now is being weaned with pressure support and 40% oxygen. Which parameters indicate that he is ready to be extubated?a. Tidal volume 500 mL; respiratory rate 22 breaths/min; patient can lift head off pillow b. Tidal volume 700 mL; respiratory rate 10 breaths/min; patient cannot lift head off pillow c. Tidal volume 500 mL; respiratory rate 6 breaths/min; patient not responsive to commandsd. Tidal volume 300 mL; respiratory rate 34 breaths/min; patient diaphoretic and restless

Correct answer: aRationale: Extubation criteria include tidal volume of at least 5 mL/kg, vital capacity of at least 10 mL/kg, maximal inspiratory pressure (also referred to as negative inspiratory force or pressure) of at least (more negative than) −20 cm H2O, and acceptable arterial blood gases. The patient also must be capable of maintaining his airway. Only option a meets the criteria.Test-Taking Strategy: Look at the patient description. Options b, c, and d clearly show patients unable to maintain an open airway and ventilation.<System> Pulmonary<Section> Mechanical Ventilation<Level> Application/Analysis

90. Which of the following would not be included in the treatment of flail chest and other bony thoracic injuries?a. Chest tubeb. Oxygen therapy c. Mechanical ventilation d. Pain management

Correct answer: aRationale: Flail chest occurs when two or more ribs are fractured in two or more locations, there is a fractured sternum, or there is a poorly healed sternotomy. Treatment of flail chest includes pain management, internal stabilization using mechanical ventilation, and oxygen to maintain the arterial oxygen saturation at a minimum of 95%. Chest tubes are used for pneumothorax, hemothorax, or pleural effusion. Patients with flail chest or rib fractures sometimes may have a concurrent pneumothorax.Test-Taking Strategy: Do not assume information that is not given. The question does not say there is a pneumothorax, so do not assume there is a pneumothorax, even though patients with a flail chest or rib fractures may have a concurrent pneumothorax. <System> Pulmonary<Section> Chest Tube<Level> Application/Analysis

65. Which mode of mechanical ventilation requires the nurse to monitor closely for auto- PEEP (positive end-expiratory pressure)?a. Inverse ratio ventilationb. Pressure support ventilation c. Intermittent mandatory ventilation d. High-frequency jet ventilation

Correct answer: aRationale: Inverse ratio ventilation allows more time for inspiration than for expiration. A potential danger of this mode is that expiration will be incomplete when the next inspiration begins. This means that there was unintentional positive end-expiratory pressure, also referred to as auto-PEEP.Test-Taking Strategy: Notice that options b, c, and d are inspiratory maneuvers. Only option a makes a change in expiration by changing the normal inspiration/expiration ratio. Because PEEP is positive end-expiratory pressure, choose option a.<System> Pulmonary<Section> Mechanical Ventilation<Level> Application/Analysis

68. A patient arrives in the emergency department after a penetrating injury to the left side of his chest. The patient is extremely dyspneic, and chest x-ray film confirms pneumothorax. Oxygen via nasal cannula is at 5 L/min. What changes in physical status would indicate an impending crisis? a. Jugular venous distentionb. Tracheal shift to the affected sidec. Chest paind. Diminished breath sounds over the left side of the chest

Correct answer: aRationale: Jugular venous distention in this patient is an indication of tension pneumothorax. Chest pain and diminished breath sounds over the affected side would be expected with pneumothorax. The trachea would be shifted to the unaffected side in tension pneumothorax.Test-Taking Strategy: Divide the options into expected with pneumothorax or indicative of a crisis, such as tension pneumothorax. Choose option a.<System> Pulmonary<Section> Thoracic Trauma<Level> Application/Analysis

88. Which of the following is not a likely cause of metabolic acidosis? a. Nasogastric suctionb. Diarrheac. Shock d. Renal failure

Correct answer: aRationale: Metabolic acidosis can be caused by loss of bicarbonate or accumulation of metabolic (nonvolatile) acids. First, consider the gastrointestinal tract. Remember that the stomach is acidic, but the gastrointestinal tract below the stomach is alkaline. Losses from the stomach cause metabolic alkalosis. Diarrhea causes loss of bicarbonate and metabolic acidosis. Shock causes accumulation of lactic acid, and renal failure causes the accumulation of metabolic acids normally excreted by the kidney. These metabolic acid accumulations cause metabolic acidosis.Test-Taking Strategy: With the first two options, consider what is lost. With the second two options, consider what is gained. The only option that would not result in bicarbonate loss or metabolic acid gain is option a, nasogastric suction.<System> Pulmonary<Section>Arterial Blood Gas<Level> Knowledge/Comprehension

77. What is the most common cause of hypoxemia in the surgical patient? a. Microatelectasisb. Pulmonary embolismc. Aspiration d. Pneumonia

Correct answer: aRationale: Microatelectasis occurs during surgery as hypoventilation occurs. Inspiratory maneuvers (e.g., deep breathing and incentive spirometry) after surgery are the best preventive measures. Note that high abdominal incisions, flank incisions, and thoracic incisions perpetuate hypoventilation because breathing is painful. Adequate analgesia also is important in prevention of atelectasis.Test-Taking Strategy: Time frame is important in this question. Anesthesia may cause hypoinflation and atelectasis. Flank incision causes pain and splinting to reduce the pain and may cause atelectasis. Choose option a.<System> Pulmonary<Section> Acute Respiratory Failure<Level> Knowledge/Comprehension

82. What is the priority of management in respiratory acidosis?a. Improve alveolar ventilation by treating the cause of hypoventilation. b. Buffer the acid with sodium bicarbonate.c. Improve oxygenation by administering oxygen.d. Decrease anxiety to increase PaCO2 level.

Correct answer: aRationale: Respiratory acidosis is caused by an increase in PaCO2. The treatment is to decrease the PaCO2 by improving ventilation. This may include bronchial hygiene techniques, deep breathing, bronchodilators, antibiotics, narcotics for pain relief, naloxone for opiate overdosage, artificial airways, mechanical ventilation, and others as indicated. Test-Taking Strategy: For respiratory acid-base imbalances, act on the respiratory system. Bicarbonate is used for severe metabolic acidosis. It may worsen respiratory acidosis because it increases carbon dioxide production. Eliminate option b. Oxygen may be necessary for patients with respiratory acidosis, but only if they are truly hypoxemic. Oxygen does not improve ventilation; it improves oxygenation. Eliminate option c. Decreasing anxiety is always important, but increasing PaCO2 levels is the treatment for respiratory alkalosis. Eliminate option d. Respiratory acidosis is treated through improvement of alveolar ventilation. Choose option a.<System> Pulmonary<Section> Arterial Blood Gas; Acute Respiratory Failure<Level> Application/Analysis

55. A decrease in static compliance would occur in which of the following? a. Pneumothorax b. Flail chest c. Bronchospasm d. Mucous plug

Correct answer: aRationale: Static compliance is the compliance of the lung when no air is moving, so it reflects the compliance of the lung and the chest wall. Dynamic compliance is the compliance of the lung when air is moving, so it reflects the compliance of the lung and chest wall plus airway resistance. Pneumothorax causes a sudden decrease in static compliance, whereas acute respiratory syndrome causes a gradual decrease in static compliance. Bronchospasm or mucous plug causes a decrease in dynamic compliance because it affects airway resistance. Flail chest actually would increase the static compliance because of the loss of intactness of the bony thorax.Test-Taking Strategy: Remember static means stasis, so air is not moving, and dynamic means moving and changing. When the lung is static and no air is moving, the compliance (stretchability) of the lung and chest wall is evaluated. When the lung is dynamic and air is moving, the compliance of the lung and chest wall and airway resistance are evaluated. Restrictive conditions affect static and dynamic compliance. Obstructive conditions affect only dynamic compliance. Pneumothorax is restrictive. Bronchospasm and mucous plugs are obstructive. Flail chest increases compliance. Choose option a.<System> Pulmonary<Section> Acute Respiratory Failure<Level> Knowledge/Comprehension

16. A 56-year-old man was admitted to the intensive care unit with a diagnosis of status asthmaticus. What is the definition of status asthmaticus?a. An asthma episode that has failed to improve with conventional therapyb. An asthma episode that is complicated by pulmonary edema or heart failure c. The first episode of asthma before any treatment has been administered d. The terminal stages of asthma

Correct answer: aRationale: Status asthmaticus is the term used for an asthma "attack" that has not responded to conventional therapy. The time frame of 24 hours is the usual standard. Test-Taking Strategy: Simply remember what status means: an unremitting state or condition. It is not terminal but unremitting. This makes option a the logical answer. <System> Pulmonary<Section> Asthma<Level> Knowledge/Comprehension

18. A 56-year-old man was admitted to the intensive care unit with a diagnosis of status asthmaticus. Which of the following conditions exacerbates dehydration in status asthmaticus?a. Tachypnea causing increase in insensible loss b. Dyspnea causing a decrease in fluid intakec. Sympathetic nervous system stimulation causing diuresis d. Respiratory infection causing fever and diaphoresis

Correct answer: aRationale: Tachypnea causes a significant increase in the sensible loss via the respiratory system. This is the most significant of the factors listed.Test-Taking Strategy: Eliminate option d because respiratory infection is not always present in status asthmaticus, but tachypnea is always present. Option b could cause dehydration long-term, but status asthmaticus is an acute process causing an active loss rather than simply a decrease in fluid intake. Although there is sympathetic nervous stimulation during physiologic stress such as in status asthmaticus, this does not cause a significant fluid volume loss. Option a is the best choice.<System> Pulmonary<Section> Asthma<Level> Application/Analysis

38. A 72-year-old woman is admitted to the critical care unit in acute respiratory distress. Her respiratory rate is 38 breaths/min and labored. Her blood pressure is 94/68 mm Hg and heart rate is 116 beats/min. The monitor shows atrial fibrillation. Lung auscultation reveals crackles in the lung bases. Ventilation-perfusion scan results indicate a high probability of pulmonary embolism. Which of the following is the most likely location for the formation of a blood clot causing a pulmonary embolism? a. In the veins of the leg b. In the left ventriclec. In the right ventricled. In the arteries of the leg

Correct answer: aRationale: The most likely location for the formation of a blood clot causing a pulmonary embolism is in the deep veins of the legs. In the deep veins of the leg, circulation is slow and blood may pool, especially in inactive or immobile persons. Blood normally travels too rapidly in the arteries and in the ventricles. Mural thrombi may occur in fibrillating atria or ventricles with aneurysmal dilation.Test-Taking Strategy: Only clots from the venous side of the heart would become pulmonary emboli, so eliminate options b and d. Clots in the right ventricle (e.g., mural thrombi associated with right ventricular aneurysm) are much less likely than clots from the deep veins in the legs.<System> Pulmonary<Section> Pulmonary Embolism<Level> Application/Analysis

12. A 70-kg male patient with acute respiratory failure is being weaned from mechanical ventilation using the intermittent mandatory ventilation (IMV) method. Ventilator settings and current arterial blood gas results are as follows: Ventilator Settings Fraction of inspired oxygen, 0.35 Tidal volume, 700 mLIMV, 4 breaths/min Arterial Blood Gases pH, 7.29HCO3, 22 mEq/L PaCO2, 52 mm Hg PaO2, 96 mm Hg Which of the following adjustments should be anticipated? a. Increase the IMV rate.b. Increase the tidal volume.c. Decrease the tidal volume. d. Decrease the IMV rate.

Correct answer: aRationale: The respiratory acidosis indicates that ventilation needs to be increased. This can be done by increasing the tidal volume or increasing the rate. Remember that tidal volume on mechanical ventilators usually is set for tidal volumes of 5 to 10 mL/kg and slow rates. Because the tidal volume is already 700 mL (10 mL/kg for a 70-kg man), increasing the tidal volume further may increase the incidence of barotrauma. In this patient, it would be preferable to increase the rate.Test-Taking Strategy: Think step by step. PaCO2 reflects ventilation. Elevated PaCO2 reflects inadequate ventilation. Minute ventilation is tidal volume multiplied by rate. Therefore increasing tidal volume or rate will increase ventilation and decrease PaCO2, so eliminate options c and d. The decision between options a and b is made by considering other factors such as risks. Increasing rate will not increase risk, but increasing tidal volume may increase risk (barotrauma). Choose option a.<System> Pulmonary<Section> Mechanical Ventilation<Level> Synthesis/Evaluation

61. A 55-year-old man is admitted to the critical care unit with a diagnosis of acute respiratory failure. This patient needs to be intubated. Which action would best assure the nurse of proper positioning of an endotracheal tube?a. Listen for bilateral breath sounds.b. Check for bilateral chest expansion.c. Listen for air over the stomach.d. Check to see whether mucus can be aspirated during suctioning of the tube.

Correct answer: aRationale: The tube fogs as it is placed into the trachea, and bilateral chest expansion should be seen with breaths delivered with a manual resuscitation bag. The most objective bedside indication of proper position of an endotracheal tube is bilateral breath sounds. If the tube is in the right mainstem bronchus (where it preferentially goes when positioned down too far), breath sounds will not be heard on the left. Chest x-ray film also should be used to ensure that the tip of the tube is 3 to 5 cm above the carina. A carbon dioxide indicator also may be used to check that the trachea, rather than the esophagus, has been intubated.Test-Taking Strategy: Look for the answer that ensures that air is getting into the lungs. Bilateral breath sounds would be the best indication that air is entering both lungs. Choose option a.<System> Pulmonary<Section> Airway<Level> Application/Analysis

79. A patient with a triple-lumen subclavian catheter has been receiving total parenteral nutrition, maintenance fluids, and antibiotics by the catheter. He has been slightly confused. Suddenly he grasps the catheter and pulls it out. He then complains of shortness of breath, and his pulse oximetry indicates an SpO2 of 84%. How should this patient be positioned? a. Head down, left sideb. Head down, right sidec. Head of bed elevated, left side d. Head of bed elevated, right side

Correct answer: aRationale: This patient most likely has developed an air embolism. Place him immediately in a head-down position on his left side (Durant's maneuver). This prevents the air embolus from entering the right ventricular outflow tract. This allows time for the air embolism to be absorbed.Test-Taking Strategy: Associate the symptoms with the recent event of the catheter disconnect. Envision a big air bubble in the patient's heart. Think: What position would decrease the movement of the air embolism out of the right side of the heart. Chose option a. <System> Pulmonary<Section> Pulmonary Embolism<Level> Synthesis/Evaluation

34. A patient had a nephrectomy yesterday. She is being monitored closely for indications of hypoventilation and atelectasis. Which of the following would be the most appropriate treatment for this patient's hypoventilation?a. Suctioning of airway as neededb. Analgesicsc. Bronchodilatorsd. Antibiotics

Correct answer: bRationale: Flank, thoracic, and high abdominal incisions often are related to atelectasis because deep breathing is painful. Adequate analgesia allows the patient to take deep breaths and perform incentive spirometry with less discomfort.Test-Taking Strategy: The case study does not identify secretions, bronchospasm, or infection as a problem. You know that a patient with a flank incision will have pain with each breath. This causes splinting and hypoventilation. Treating the pain will improve ventilation. Choose option b.<System> Pulmonary<Section> Acute Respiratory Failure; Pharmacology<Level> Application/Analysis

1. A patient is admitted to the critical care unit with acute exacerbation of chronic obstructive pulmonary disease (COPD) caused by respiratory infection. Temperature is 39° C, blood pressure is 150/82 mm Hg, heart rate is 110 beats/min, and respiratory rate is 24 breaths/min. Chest x-ray film confirms that there is left upper lobe pneumonia. Arterial blood gases reveal the following: pH 7.29 PaCO2 62 mm Hg HCO3 30 mEq/L PaO2 52 mm Hg A PaO2 of less than 50 to 60 mm Hg and a PaCO2 greater than 50 mm Hg with a pH less than 7.3 indicates which of the following? a. Obstructive lung disease b. Acute respiratory failure c. Restrictive lung disease d. Acute respiratory distress syndrome

Correct answer: b Rationale: Acute respiratory failure is defined by arterial blood gas results with PaO2 of less than 50 to 60 mm Hg and PaCO2 of greater than 50 mm Hg. In patients with COPD (like this patient), the qualifier of "with a pH less than 7.3" needs to be added because the patient chronically has a PaCO2 greater than 50 mm Hg. Acute respiratory failure with hypoxemia and without hypercapnia is classified as type I, whereas hypoxemia with hypercapnia is classified as type II. Test-Taking Strategy: Acute respiratory failure is a more general term. Obstructive lung disease, restrictive lung disease, and acute respiratory distress syndrome are more specific. If you are not sure about the correct answer and there is one general answer and three specific answers, choose the most general because it is more likely to be correct. <System> Pulmonary <Section> Acute Respiratory Failure <Level> Knowledge/Comprehension

2. A patient is admitted to the critical care unit with acute exacerbation of chronic obstructive pulmonary disease caused by respiratory infection. Temperature is 39° C, blood pressure is 150/82 mm Hg, heart rate is 110 beats/min, and respiratory rate is 24 breaths/min. Chest x-ray film confirms that there is left upper lobe pneumonia. Arterial blood gases reveal the following: pH 7.29 PaCO2 62 mm Hg HCO3 30 mEq/L PaO2 52 mm Hg Collaborative management for this patient should include which of the following? a. Intubation, mechanical ventilation, and positive end-expiratory pressure b. Bronchial hygiene, antibiotics, and oxygen c. Oxygen, bronchodilators, and steroids d. Bronchial hygiene, oxygen, and bicarbonate

Correct answer: b Rationale: Collaborative management of type II acute respiratory failure is treatment of cause plus modalities to improve ventilation. Bronchial hygiene measures (e.g., coughing, suctioning, and postural drainage) to mobilize secretions and antibiotics to treat infection are needed to treat the hypoventilation, whereas oxygen is required to treat the moderate degree of hypoxemia. Remember that improving ventilation will improve oxygenation, but oxygen therapy may be needed acutely. Test-Taking Strategy: Because the case study clearly states that there is an infection, look for antibiotics in the answer. Option b is the only one with antibiotics, and the other methods listed with it are logical in the treatment of respiratory infection. In addition, remember "simple to complex." Do not intubate and mechanically ventilate a patient until more conservative methods have failed or are likely to be ineffective because of respiratory muscle fatigue or respiratory arrest. Bronchodilators (option c) can be eliminated by the fact that no wheezing or bronchospasm are identified in the case study. Bicarbonate (option d) is indicated only for severe metabolic acidosis, not respiratory acidosis. <System> Pulmonary <Section> Acute Respiratory Failure; Pharmacology

5. A patient is admitted to the critical care unit with acute exacerbation of COPD caused by respiratory infection. Temperature is 39° C, blood pressure is 150/82 mm Hg, heart rate is 110 beats/min, and respiratory rate is 24 breaths/min. Chest x-ray film confirms that there is left upper lobe pneumonia. Arterial blood gases reveal the following: pH 7.29 PaCO2 62 mm Hg HCO3 30 mEq/L PaO2 52 mm Hg Which assessment findings would the nurse expect over the area of pneumonia in the left upper lobe in this patient? a. Resonance to percussion, bronchial breath sounds b. Dullness or flatness to percussion, bronchial breath sounds c. Resonance to percussion, vesicular breath sounds d. Tympany to percussion, vesicular breath sounds

Correct answer: b Rationale: Pneumonia acts as a solid mass on assessment as purulent material in the alveolus consolidates. Consolidation causes dullness to percussion and bronchial (tubular sounding) breath sounds on auscultation. Test-Taking Strategy: Look for and eliminate normal because pneumonia is not normal. Resonance to percussion (options a and c) and vesicular breath sounds (options c and d) are normal. This leaves only option b. <System> Pulmonary <Section> Pneumonia <Level> Knowledge/Comprehension

6. A 24-year-old man is admitted to the critical care unit after sustaining a pulmonary contusion in a motor vehicle collision. He has no history of cardiac or pulmonary disease. During the first 24 hours after admission, he has been complaining of increasing dyspnea, his respiratory rate has been increasing, and his oxygen saturation via pulse oximetry has been decreasing despite supplemental oxygen. Breath sound assessment reveals fine crackles bilaterally. Arterial blood gases reveal respiratory alkalosis and hypoxemia. Chest x-ray film reveals patchy infiltrates. Acute respiratory distress syndrome is diagnosed. Oxygen therapy is initiated, and arterial blood gases are monitored closely, but SaO2 continues to fall. Which of these oxygen delivery systems will provide the highest concentration of oxygen? a. Face tent b. Non-rebreathing mask c. Nasal cannula d. Venturi mask

Correct answer: b Rationale: The non-rebreathing mask stores oxygen in nose, pharynx, mask, and reservoir bag between breaths. This allows a concentration of close to 100%. Test-Taking Strategy: Count reservoirs: The more reservoirs, the higher the concentration. Consider the nose and pharynx as one reservoir together. A mask would be another reservoir. A reservoir bag would be another reservoir. So consider the oxygen delivery systems. Nasal cannula: nose and pharynx only (one). Face tent and Venturi mask: nose and pharynx and tent or mask (two). Non-rebreathing mask: nose and pharynx, mask, reservoir bag (three). Choose option b. <System> Pulmonary <Section> Oxygen <Level> Knowledge/Comprehension

53. A patient with acute respiratory distress syndrome develops a pneumothorax resulting from high levels of positive end-expiratory pressure. Which clinical manifestations would indicate pneumothorax?a. Crackles in lung bases and dyspnea b. Increase in peak inspiratory pressure and unequal breath soundsc. Pleural friction rub on affected side and decrease in venous oxygen saturation d. Chest pain and rhonchi through lung fields

Correct answer: bRationale: A sudden increase in peak inspiratory pressure and diminished breath sounds on the affected side are indications of pneumothorax. When there is air in the pleural space, the normal negative pressure in the pleural space is lost and the lung collapses. Compliance of the collapsed lung decreases dramatically, so more pressure is required to inflate the lung, so peak inspiratory pressure increases. Diminished breath sounds over the affected side are caused by air between the lung and the chest wall and decreased ventilation.Test-Taking Strategy: Crackles are associated with atelectasis, pulmonary edema, or pulmonary fibrosis, so eliminate option a. Pleural friction rub is associated with pleurisy or pulmonary infarction, so eliminate option c. Rhonchi are associated with mucus or fluid in the airways, so eliminate option d. Breath sounds are decreased over the affected side, so choose option b.<System> Pulmonary<Section> Thoracic Trauma; Mechanical Ventilation<Level> Knowledge/Comprehension

100. A patient is admitted with pneumonia and has a temperature of 40° C (104° F). His PaO2 is 60 mm Hg. His arterial oxygen saturation (SaO2) would be expected to be approximately:a. 90%. b. 86%. c. 93%. d. 75%.

Correct answer: bRationale: Hyperthermia shifts the oxyhemoglobin dissociation curve to the right, which decreases the affinity between oxygen and hemoglobin. Normally a PaO2 of 60 mm Hg would be correlated to an SaO2 of 90%, but his would be less because of the shift to the right.Test-Taking Strategy: If you remember that hyperthermia shifts the curve, and you know that a PaO2 of 60 mm Hg usually is correlated to an SaO2 of 90%, then you narrowed your choices to options b and c.<System> Pulmonary<Section> Hypoxia<Level> Synthesis/Evaluation

60. A 55-year-old man is admitted to the critical care unit with a diagnosis of acute respiratory failure. Which of the following would be expected in this patient's arterial blood gas results?a. PaO2 less than 80 mm Hg b. PaO2 less than 60 mm Hg and/or PaCO2 greater than 50 mm Hgc. PaCO2 greater than 50 mm Hg in a patient with chronic obstructive pulmonary disease d. pH less than 7.35

Correct answer: bRationale: Acute respiratory failure frequently is defined by arterial blood gas abnormality. PaO2 is less than 50 to 60 mm Hg and/or the PaCO2 is greater than 50 mm Hg. Keep in mind that in patients with chronically elevated PaCO2 (e.g., COPD), the pH additionally must be less than 7.3 because the patient must have decompensated to be in acute respiratory failure. A PaO2 of less than 80 mm Hg is by definition hypoxemia (decrease in oxygen in the blood), but it is not severe enough to be considered acute respiratory failure. Test-Taking Strategy: Associate acute respiratory failure with "50-50." The PaO2 is less than 50 to 60 mm Hg, and/or PaCO2 is greater than 50 mm Hg. Choose option b. <System> Pulmonary<Section> Acute Respiratory Failure<Level> Knowledge/Comprehension

64. A 65-year-old man is admitted with chronic obstructive pulmonary disease (COPD) and pulmonary embolism. Which of the following complications of COPD predispose the patient to the development of pulmonary embolism?a. Atrial fibrillation and right ventricular hypertrophyb. Atrial fibrillation and polycythemiac. Pulmonary hypercapnia and polycythemiad. Pulmonary hypertension and right ventricular hypertrophy

Correct answer: bRationale: COPD causes pulmonary hypertension, right ventricular hypertrophy, and right atrial enlargement. The right atrial enlargement causes stretching of the atrial tissue and frequently results in atrial dysrhythmias. Atrial fibrillation is associated with mural thrombi and pulmonary and systemic emboli. The hypoxemia that results from COPD causes release of erythropoietin from the kidney, which stimulates release of red blood cells from the bone marrow and polycythemia. Polycythemia causes hypercoagulability. This hypercoagulability, especially coupled with inactivity, increases the risk of thrombosis and emboli.Test-Taking Strategy: Remember that emboli start as clots. Look for options that include factors that increase the risk of clotting. Pulmonary hypertension and right ventricular hypertension do not cause clotting, but atrial fibrillation and polycythemia do. Choose option b.<System> Pulmonary<Section> Pulmonary Embolism<Level> Synthesis/Evaluation

63. A 55-year-old man is admitted to the critical care unit with diagnosis of acute respiratory failure. He has been on mechanical ventilation with assist-control for 2 weeks. If this patient meets weaning criteria, which weaning technique probably would be used? a. T-pieceb. Intermittent mandatory ventilation (IMV)c. Continuous positive airway pressure (CPAP)d. Assist-control

Correct answer: bRationale: Considering the long period this patient has been on a ventilator, IMV will allow gradual weaning. This will allow the patient gradually to resume responsibility for ventilation while his ventilatory muscle strength increases.Test-Taking Strategy: The important point in this question is the time frame. He has been on a ventilator for 2 weeks. He needs time gradually to take over ventilation from the machine. T-piece and assist-control are the two extremes. With the T-piece method, he has supplemental oxygen support but no ventilatory support. With assist-control, even the triggered breaths are assisted. This does not allow adequate muscle strengthening. CPAP supports oxygenation but does not provide the mechanical support this patient needs at least initially. Choose IMV, option b, as the most reasonable method for this patient.<System> Pulmonary<Section> Mechanical Ventilation<Level> Application/Analysis

78. A 48-year-old male patient admitted with acute respiratory failure has just extubated himself. He now has inspiratory stridor and hoarseness. Along with reassurance and humidified oxygen, what other intervention is indicated for this patient at this time?a. Bronchodilators b. Corticosteroids c. Antibioticsd. Diuretics

Correct answer: bRationale: Corticosteroids are used to reduce mucosal swelling.Test-Taking Strategy: Note that there is stridor, not wheezing, so eliminate option a. Infection is not mentioned, so eliminate option c. Diuretics would decrease circulating volume but would not reduce edema locally in the trachea. Choose option b. <System> Pulmonary<Section> Acute Respiratory Failure; Pharmacology<Level> Application/Analysis

84. Which of the following is the best indicator of balance between oxygen delivery and oxygen consumption?a. SaO2b. SvO2 c. DO2 d. VO2

Correct answer: bRationale: DO2 is oxygen delivery. VO2 is oxygen consumption. SaO2 is one component of oxygen delivery (the other two components being hemoglobin and cardiac output). SvO2 is the saturation of the venous blood as it returns to the lung for reoxygenation. SvO2 is decreased if DO2 decreases or oxygen consumption increases.Test-Taking Strategy: Note the word balance. SvO2 is the balance of oxygen after the tissues have used what they need. SaO2 is a component of DO2, so both affect delivery. VO2 is tissue consumption, and SvO2 is what is left after delivery and consumption (balance). Choose option b.<System> Pulmonary<Section> Hypoxia<Level> Application/Analysis

62. A 55-year-old man is admitted to the critical care unit with a diagnosis of acute respiratory failure. The patient must be ventilated mechanically for 2 weeks. Initial weaning efforts are to be initiated tomorrow. Which of the following would improve the patient's chance for successful weaning? a. High protein, high carbohydrate, low fat b. High protein, low carbohydrate, high fat c. Low protein, high carbohydrate, low fat d. Low protein, low carbohydrate, high fat

Correct answer: bRationale: Digestion of carbohydrates produces more carbon dioxide than does digestion of proteins and fats. The diet should be high in protein and high in calories, but the calories should be in the form of fats rather than carbohydrates. Pulmocare brand enteral feeding is high in protein and fat and low in carbohydrates. The same adjustment can be made if the patient is receiving parenteral feedings by decreasing the dextrose amount and increasing the amount of Intralipid.Test-Taking Strategy: Protein is increased in most critical care conditions except in hepatic encephalopathy and renal failure. Eliminate options c and d, which include low protein. Calories must be increased to prevent the patient from using protein or lean body mass for energy. In pulmonary patients, these calories should be in the form of fats to prevent excess carbon dioxide production from carbohydrate metabolism, especially during weaning. Choose option b.<System> Pulmonary<Section> Mechanical Ventilation<Level> Synthesis/Evaluation

39. Which risk factor is associated most commonly with aspiration? a. Impaired mobilityb. Impaired consciousnessc. Febrile state d. Hypoventilation

Correct answer: bRationale: Impaired consciousness is the most common risk factor associated with aspiration. If consciousness is impaired, the person is not aware of fluid or particulate matter that has entered the airway. This can lead to pulmonary infections or airway obstruction.Test-Taking Strategy: Only in impaired consciousness are the normal airway protective mechanisms impaired. Choose option b.<System> Pulmonary<Section> Aspiration<Level> Knowledge/Comprehension

85. Which of the following is an important component of care for patients receiving mechanical ventilation with inverse ratio ventilation (IRV) mode?a. Frequent suctioningb. Sedation c. Bilateral chest tubes d. Diuretics

Correct answer: bRationale: In IRV, the normal inspiratory-to-expiratory ratio is reversed. Because this is unnatural, it produces anxiety for the patient. Sedation (e.g., propofol) or paralysis is required. Suctioning is performed only as indicated. Although IRV does cause auto-positive end-expiratory pressure and is associated with an increased incidence of barotrauma, chest tubes are placed only if pneumothorax occurs. Diuretics would accentuate the decrease in preload and hemodynamic consequences caused by mechanical ventilation.Test-Taking Strategy: Consider that mechanical ventilation (especially in IRV mode) is anxiety producing. Sedation certainly would be indicated.<System> Pulmonary<Section> Mechanical Ventilation<Level> Synthesis/Evaluation

69. A patient arrives in the emergency department after a penetrating injury to the left side of his chest. The patient is extremely dyspneic, and chest x-ray film confirms pneumothorax. Oxygen via nasal cannula is at 5 L/min. A pleural chest tube is inserted. While assessing the water-seal drainage system, the nurse notes continuous bubbling in the water-seal chamber. What does this indicate? a. A patent systemb. An air leakc. A clot or kink in systemd. An excessive amount of suction

Correct answer: bRationale: Intermittent bubbling in the water-seal chamber is expected until the lung is reexpanded. Continuous bubbling means that there is a continuous air leak such as in the system or a bronchopleural fistula. A patent system would fluctuate with ventilation and have an occasional bubble indicative of air escaping from the pleural space through the water-seal chamber column of fluid. A clot or kink in the system would result in absence of fluctuation in the water-seal chamber with ventilation. Excessive suction would cause continuous, excessive bubbling in the suction control chamber.Test-Taking Strategy: Bubbling is air, so eliminate option c. Continuous bubbling is not normal, so eliminate option a. Excessive suction would affect the suction control chamber, not the water-seal chamber, so eliminate option d. Choose option b.<System> Pulmonary<Section> Chest Tube<Level> Application/Analysis

9. A 24-year-old man is admitted to the critical care unit after sustaining a pulmonary contusion in a motor vehicle collision. He has no history of cardiac or pulmonary disease. Over the last few hours he has been complaining of increasing dyspnea, his respiratory rate has been increasing, and his oxygen saturation via pulse oximetry has been decreasing. Breath sound assessment reveals fine crackles bilaterally. Arterial blood gases reveal respiratory alkalosis and hypoxemia. Chest x-ray film reveals patchy infiltrates. Acute respiratory distress syndrome (ARDS) is diagnosed. Oxygen therapy is initiated, and arterial blood gases are monitored closely. The massive atelectasis that occurs in acute respiratory distress syndrome is a classic example of intrapulmonary shunt. Intrapulmonary shunting is best described as: a. ventilated alveoli having blocked perfusion.b. perfused alveoli having blocked ventilation.c. decreased diffusion caused by an increased thickness of the alveolar-capillary membrane.d. decreased diffusion caused by decreased driving pressure of oxygen.

Correct answer: bRationale: Intrapulmonary shunt occurs when blood comes past nonventilated alveoli and goes back to the left side of the heart without its normal oxygen load. This sometimes is described as low ventilation/perfusion ratio.Test-Taking Strategy: Relate back to intracardiac shunt, as in "blue babies." In this case the blood goes from the right side of the heart to the left side of the heart without going through the lungs. In intrapulmonary shunt, the pathway is longer, but the end result is the same: dramatically reduced oxygen saturation.<System> Pulmonary<Section> Acute Respiratory Failure<Level> Knowledge/Comprehension

10. A 24-year-old man is admitted to the critical care unit after sustaining a pulmonary contusion in a motor vehicle collision. He has no history of cardiac or pulmonary disease. Over the last few hours he has been complaining of increasing dyspnea, his respiratory rate has been increasing, and his oxygen saturation via pulse oximetry has been decreasing. Breath sound assessment reveals fine crackles bilaterally. Arterial blood gases reveal respiratory alkalosis and hypoxemia. Chest x-ray film reveals patchy infiltrates. Acute respiratory distress syndrome (ARDS) is diagnosed. Oxygen therapy is initiated, and arterial blood gases are monitored closely. Intrapulmonary shunt volume can be estimated by measuring: a. arterial-venous oxygen difference. b. alveolar-arterial oxygen difference. c. PaCO2.d. venous oxygen saturation (SvO2).

Correct answer: bRationale: Intrapulmonary shunt results in a difference between the amount of oxygen in the alveolus and the amount of oxygen that gets into the arterial blood. A change in the arterial-venous oxygen difference would indicate a change in tissue oxygen consumption. A change in PaCO2 indicates a change in ventilation. A change in SvO2 could indicate a change in oxygen delivery (arterial oxygen saturation, hemoglobin, cardiac output) or a change in tissue oxygen consumption.Test-Taking Strategy: Look for the answers that are lung specific. These are options b and c. PaCO2 (option c) is reflective of changes in ventilation. To come to the conclusion of option b, think about the alveolar-capillary membrane and the fact that the process of diffusion is between the alveolus and the arterial blood.<System> Pulmonary<Section> Acute Respiratory Failure<Level> Application/Analysis

92. Which of the following accurately describes the paradoxical chest wall movement associated with a flail chest?a. Chest wall segment moving outward on inspiration and inward on expirationb. Chest wall segment moving inward on inspiration and outward on expiration c. Chest wall moving outward on inspiration and abdominal wall moving inward d. Chest wall moving inward on inspiration and abdominal wall moving outward

Correct answer: bRationale: The flail segment moves in the direction opposite from normal chest excursion. The chest normally moves out during inspiration, but the flail segment moves inward. The chest normally moves in during expiration, but the flail segment moves outward.Test-Taking Strategy: None<System> Pulmonary<Section> Thoracic Trauma<Level> Application/Analysis

25. A 54-year-old man has just returned to the critical care unit from the postanesthesia care unit. He has a 60-pack-year history of cigarette smoking and had a right lower lobectomy performed earlier today for treatment of lung cancer. He is still intubated and on a positive pressure mechanical ventilator. Considering the patient's smoking history, a consistent and reliable fraction of inspired oxygen (FiO2) is desired. Which of the following systems would be best for this patient and give the most reliable FiO2 administration? a. Nasal cannulab. Venturi maskc. Rebreathing mask d. Face mask

Correct answer: bRationale: Nasal cannula, rebreathing mask, and face mask are low-flow oxygen delivery systems, which do not ensure a reliable FiO2. The Venturi mask is a high-flow oxygen delivery system that does ensure a reliable FiO2 and provides the lower FiO2 levels necessary for patients with chronic hypercapnia.Test-Taking Strategy: The key word in this question is reliable. Low-flow oxygen delivery systems do not ensure a reliable oxygen concentration because the amount of diluent (room air) inspired varies with ventilatory rate and depth.<System> Pulmonary<Section> Oxygen<Level> Application/Analysis

35. A 72-year-old woman is admitted to the critical care unit in acute respiratory distress. Her respiratory rate is 38 breaths/min, deep and labored. Her blood pressure is 94/68 mm Hg and heart rate is 116 beats/min. The monitor shows atrial fibrillation. Lung auscultation reveals crackles in the lung bases. Ventilation-perfusion scan results indicate a high probability of pulmonary embolism. Which of the following reflects the expected arterial blood gases? a. Decreased pH, increased PaCO2, and decreased PaO2 b. Increased pH, decreased PaCO2, and decreased PaO2 c. Decreased pH, increased PaCO2, and normal PaO2d. Increased pH, decreased PaCO2, and normal PaO2

Correct answer: bRationale: Patients with pulmonary embolus experience dyspnea and tachypnea. This causes respiratory alkalosis (pH increased, PaCO2 decreased). Hypercapnia and acidosis eventually may occur as the patient fatigues, but this patient has a respiratory rate of 32 breaths/min and deep (i.e., hyperpnea). PaO2 is decreased because of perfusion defect and ventilation/perfusion mismatch.Test-Taking Strategy: Note that the case study identifies respirations that are deep at a rate of 38 breaths/min. This likely would cause a respiratory alkalosis, so eliminate options a and c. You know that this patient would be hypoxemic because of the ventilation-perfusion mismatch, so choose option b.<System> Pulmonary<Section> Pulmonary Embolism<Level> Application/Analysis

26. A 54-year-old man has just returned to the critical care unit from the postanesthesia care unit. He has a 60-pack-year history of cigarette smoking and had a right lower lobectomy performed earlier today for treatment of lung cancer. He is still intubated and on a positive pressure mechanical ventilator. Which of the following would be the best position for this patient to optimize ventilation and perfusion? a. On his operative side or backb. On his nonoperative side or backc. Proned. From nonoperative side to operative side

Correct answer: bRationale: Putting the operative lung up encourages ventilation and reexpansion of the operative lung. Putting the nonoperative lung down enhances blood flow to the "good lung." Turning should still be performed but from the nonoperative lung to the back during the initial recovery phase. Prone position is recommended for acute respiratory distress syndrome. Prone is not a position used for postthoracotomy patients.Test-Taking Strategy: Remember "good lung down." This applies to surgical or medical unilateral lung conditions.<System> Pulmonary<Section> Thoracotomy<Level> Application/Analysis

80. What is the goal of oxygen therapy in a hypoxemic patient with chronic obstructive pulmonary disease (COPD)?a. Prevent dyspneab. Maintain arterial oxygen saturation (SaO2) at ~90% c. Avoid more than 2 L by nasal cannula d. Prevent cyanosis

Correct answer: bRationale: Remember than patients who are chronically hypercapnic (e.g., in COPD) depend on peripheral chemoreceptors detecting hypoxemia. If this hypoxemia is eliminated, the patient's stimulus to breathe is eliminated. The patient's low SaO2 and PaO2 provide that stimulus, not the fraction of inspired oxygen. Thus oxygen can be given safely until the SaO2 is approximately 90% or the PaO2 is 60 mm Hg. Below this level, the tissues are hypoxic, but levels significantly above this may eliminate the drive. Cyanosis is hemoglobin dependent. Many of these patients are polycythemic and are cyanotic all the time (this is the source of the term blue bloaters). Elimination of dyspnea may not be possible without elimination of the hypoxic drive.Test-Taking Strategy: Dyspnea and cyanosis are not as specific as SaO2 of ~90%. To narrow your choice to option b, you must forget previously learned "rules" and understand physiology.<System> Pulmonary<Section> Acute Respiratory Failure<Level> Application/Analysis

99. A patient admitted with an acute upper gastrointestinal bleed has a blood pressure of 104/64 mm Hg, a heart rate of 110 beats/min, and a respiratory rate of 28 breaths/min. The most recent hemoglobin value is 8 g/dL. Which of the following treatments will be most important in improving his oxygen delivery (DO2)? a. Isotonic fluidsb. Packed red blood cellsc. Dobutamined. Supplemental oxygen therapy

Correct answer: bRationale: Remember that DO2 is affected by SaO2, cardiac output, and hemoglobin. In this patient, administering blood (i.e., hemoglobin) is crucial to increasing the DO2. Test-Taking Strategy: Always think first of treating the cause. He lost blood, so he needs blood.<System> Pulmonary<Section> Hypoxia; Pharmacology<Level> Synthesis/Evaluation

98. Which of the following is the most effective method of preventing postoperative atelectasis?a. Coughingb. Sustained inspiratory maneuvers c. Intermittent positive pressure breathing (IPPB) d. Bronchodilators

Correct answer: bRationale: Techniques that encourage sustained inspiration are most effective in preventing postoperative pulmonary complications. This would be deep breathing and incentive spirometry. Coughing should be encouraged only when the patient has rhonchi because it is forced expiration and actually may contribute to atelectasis when used routinely. Bronchodilators are used only if bronchospasm is an issue. IPPB was used in the past, but deep breathing and incentive spirometry are more effective without the barotrauma risk of IPPB.Test-Taking Strategy: In this case, the most conservative option is the most effective. <System> Pulmonary<Section> Acute Respiratory Failure<Level> Application/Analysis

76. A patient is admitted to the critical care unit for dyspnea. The patient's arterial blood gases reveal pH of 7.35, PaCO2 of 54 mm Hg, HCO3 of 28 mEq/L, and PaO2 of 62 mm Hg. What acid-base imbalance is evident?a. Uncompensated respiratory acidosis b. Compensated respiratory acidosis c. Uncompensated metabolic alkalosis d. Compensated metabolic alkalosis

Correct answer: bRationale: The pH, though normal, leans toward the acidotic end of the normal range. This indicates compensation. The PaCO2 is elevated, and HCO3 is elevated. When determining the primary disorder versus the system compensating, note the direction toward which the pH is leaning. An elevation of PaCO2 causes a respiratory acidosis, whereas an elevation of HCO3 causes a metabolic alkalosis. A compensating pH leans toward the initial disorder, so the primary disorder is respiratory acidosis, and there is full metabolic compensation because the pH is within normal limits.Test-Taking Strategy: This is a compensated disorder because the pH is normal, but both indicators are abnormal. PaCO2 is the respiratory indicator, whereas the HCO3 is the metabolic indicator. Eliminate the uncompensated disorders, options a and c. The pH leans in the direction of the initial disorder. Choose the compensated acidosis, option b. <System> Pulmonary<Section> Arterial Blood Gas<Level> Application/Analysis

24. A 54-year-old man has just returned to the critical care unit from the postanesthesia care unit. He has a 60-pack-year history of cigarette smoking and had a right lower lobectomy performed earlier today for treatment of lung cancer. He is still intubated and on a positive pressure mechanical ventilator. The next morning a short-term breathing trial is conducted. Spontaneous ventilatory parameters and arterial blood gases are measured in preparation for weaning and extubation. Which of the following ventilatory parameters are most indicative of the patient's ability to cough and clear his airways? a. Tidal volume and vital capacityb. Vital capacity and negative inspiratory pressurec. Tidal volume and minute ventilationd. Minute ventilation and maximal inspiratory pressure

Correct answer: bRationale: The patient must be able to pull air into his lungs to be able to perform a forcible cough. Being able to pull a significant negative (inspiratory) pressure and take a deep breath (vital capacity) are critical in performing an effective cough.Test-Taking Strategy: Think about how you cough: You pull in (negative pressure) a deep breath (vital capacity). Eliminate minute ventilation because coughing is an instantaneous event, not one that lasts a minute. This excludes options c and d. Tidal volume and vital capacity in option a are somewhat redundant because tidal volume is part of vital capacity, so eliminate option a. Choose option b.<System> Pulmonary<Section> Mechanical Ventilation<Level> Synthesis/Evaluation

96. Which effect does hypoxemia have on the pulmonary vasculature? a. Vasodilationb. Vasoconstrictionc. Vasospasm d. Coagulation

Correct answer: bRationale: This is called hypoxemic pulmonary vasoconstriction, and it is the most common cause of pulmonary hypertension.Test-Taking Strategy: None<System> Pulmonary<Section> Oxygen<Level> Knowledge/Comprehension

33. A patient had a nephrectomy yesterday. She is being monitored closely for indications of hypoventilation and atelectasis. If this patient is hypoxemic because of hypoventilation, which of the following is the most appropriate treatment to improve oxygenation?a. Administer oxygen.b. Treat the cause of hypoventilation.c. Initiate intubation and mechanical ventilation.d. Administer a fibrinolytic.

Correct answer: bRationale: Treatment of the cause of hypoventilation will improve oxygenation. Remember that in a patient breathing room air, hypercapnia causes hypoxemia (review Dalton's law if you still have questions about this). Therefore, if you get the PaCO2 down, the PaO2 will increase. Oxygen therapy is used if hypoxemia persists despite improvement in ventilation, but improvement in ventilation is the priority. Fibrinolytic and anticoagulant therapy may improve regional problems with perfusion (e.g., pulmonary embolus) but not problems with ventilation. Intubation and mechanical ventilation may be necessary if other methods do not improve ventilation.Test-Taking Strategy: Be logical. If the problem causing hypoxemia is hypoventilation, then the treatment of hypoventilation is the priority. Choose option b.<System> Pulmonary<Section> Acute Respiratory Failure; Pharmacology<Level> Application/Analysis

13. A 70-kg male patient with acute respiratory failure is being weaned from mechanical ventilation using the intermittent mandatory ventilation (IMV) method. Ventilator settings and current arterial blood gas results are as follows: Ventilator Settings Fraction of inspired oxygen, 0.35 Tidal volume, 700 mLIMV, 4 breaths/min Arterial Blood Gases pH, 7.29HCO3, 22 mEq/L PaCO2, 52 mm Hg PaO2, 96 mm Hg The patient has copious secretions. Before suctioning the patient's endotracheal tube, the nurse should adjust the vacuum pressure so that it is:a. as high as necessary.b. 100 mm Hg of vacuum. c. 60 mm Hg of vacuum.d. 10 mm Hg below the systolic blood pressure.

Correct answer: bRationale: Using a suction level as high as necessary certainly could increase trauma to the tracheobronchial tree. Pressure as low as 60 mm Hg would be unlikely to clear copious secretions. This pressure has nothing to do with systolic blood pressure.Test-Taking Strategy: Eliminate option d because you know that suctioning the tracheobronchial tree has nothing to do with systolic blood pressure. If in doubt, choose the middle of the other options. Choose option b.<System> Pulmonary<Section> Airway<Level> Knowledge/Comprehension

95. Which of the following describes the process of ventilation?a. The movement of blood saturated with oxygen to the tissues to be used b. The movement of air into and out of the lungsc. The movement of oxygen across the alveolar-capillary membraned. The distribution of air throughout the lungs

Correct answer: bRationale: Ventilation is the movement of air into and out of the lungs. The air then is distributed throughout the lungs. The oxygen then is diffused across the alveolar-capillary membrane and attaches to the hemoglobin. Finally the heart and vascular system deliver the oxygen to the cells so that the mitochondria can use the oxygen to make cellular energy (i.e., adenosine triphosphate).Test-Taking Strategy: The process starts with ventilation, which is getting the air into the lungs.<System> Pulmonary<Section> Pulmonary Physiology<Level> Knowledge/Comprehension

3. A patient is admitted to the critical care unit with acute exacerbation of chronic obstructive pulmonary disease (COPD) caused by respiratory infection. Temperature is 39° C, blood pressure is 150/82 mm Hg, heart rate is 110 beats/min, and respiratory rate is 24 breaths/min. Chest x-ray film confirms that there is left upper lobe pneumonia. Arterial blood gases reveal the following: pH 7.29 PaCO2 62 mm Hg HCO3 30 mEq/L PaO2 52 mm Hg Which of the following statements about oxygen therapy for this patient is true? a. Oxygen therapy is not indicated for this patient at this time. b. Oxygen therapy should be implemented with a nasal cannula at a flow rate of no more than 2 L/min. c. Oxygen therapy should be implemented with a nasal cannula at a flow rate to achieve an oxygen saturation of 90% via pulse oximetry. d. Oxygen therapy should be implemented via a face mask at a flow rate to achieve an oxygen saturation of 95% via pulse oximetry.

Correct answer: c Rationale: By history (COPD), you should suspect that this patient chronically has a PaCO2 greater than 50 mm Hg and depends on peripheral chemoreceptors sensitive to low oxygen levels to stimulate breathing. Oxygen is indicated for this patient because the PaO2 is less than 55 to 60 mm Hg, and therefore this degree of hypoxemia (decreased oxygen in the blood) is likely to cause hypoxia (decreased oxygen in the tissues). Oxygen therapy should be guided by PaO2 and arterial oxygen saturation (SaO2), not by fraction of inspired oxygen (FiO2), so do not follow set rules such as illustrated by option b. Oxygen via nasal cannula can be adjusted to achieve an SaO2 of 90% via pulse oximetry. A face mask cannot give less than 40% oxygen, which would almost certainly increase the SaO2 above 95% and cause respiratory depression by eliminating the patient's hypoxic drive. Test-Taking Strategy: Remember that the blood oxygenation goal is PaO2 of 60 mm Hg, which generally correlates to a blood oxygen saturation of 90%. Remember that hemoglobin saturation increases only slightly for PaO2 levels above 60 mm Hg, but hemoglobin saturation decreases significantly below 60 mm Hg. Physiologically, this is the change in the oxyhemoglobin curve from horizontal to vertical. So oxygen is indicated (eliminate option a) but not to the level indicated in option d. Eliminate option b because it does not consider this patient; use the patient's SaO2 (not the FiO2) to guide oxygen therapy. <System> Pulmonary <Section> Acute Respiratory Failure; Oxygen

8. Which of the following vital sign changes is associated with a submassive or massive pulmonary embolism?a. Hyperthermiab. Bradycardia c. Hypotensiond. Bounding arterial pressure

Correct answer: c<Rationale>Submassive or massive pulmonary embolism causes obstruction in the pulmonary circuit, which results in impairment in left ventricular filling, often called forward failure of the left ventricle.Test-Taking Strategy: Choose the most serious because submassive or massive pulmonary embolism is often fatal. Choose hypotension.<System> Pulmonary<Section> Pulmonary Embolism<Level> Knowledge/Comprehension

57. A 66-year-old man is admitted to the critical care unit with multiple trauma. He develops acute respiratory distress syndrome (ARDS) and requires intubation and mechanical ventilation. Ventilator settings are as follows: Fraction of inspired oxygen (FiO2): 0.6 Intermittent mandatory ventilation: 10 breaths/min Tidal volume: 600 mL Positive end-expiratory pressure (PEEP): 15 cm H2O Which of the following is not an effect of PEEP in this patient? a. Increases driving pressure of oxygenb. Opens collapsed alveolic. Increases risk of oxygen toxicity d. Decreases alveolar surface tension

Correct answer: cRationale: ARDS is characterized by decreased pulmonary compliance, increased alveolar surface tension, alveolar collapse, and massive intrapulmonary shunt. Oxygen therapy is ineffective until it can get to the alveolar-capillary membrane. PEEP opens collapsed alveoli (called alveolar recruitment). By keeping the alveoli open at the end of expiration, PEEP decreases alveolar surface tension. Driving pressure is increased by the positive pressure during inspiration and expiration. This allows the achievement of a given PaO2 at a lower FiO2 or achievement of a higher PaO2 at the same FiO2. This decreases the risk of oxygen toxicity.Test-Taking Strategy: Remember that driving pressure is equal to FiO2 multiplied by the barometric pressure. So driving pressure can be increased by increasing the FiO2 or increasing the barometric pressure, such as with a hyperbaric chamber. Continuous positive airway pressure and PEEP increase the driving pressure of oxygen by increasing the amount of time that the barometric pressure is above 0 throughout inspiration and expiration. This allows you to use a lower oxygen concentration to achieve a desirable PaO2 and decreases the risk of oxygen toxicity. Note that this is a negatively stated question, so read slowly and carefully. Choose option c.<System> Pulmonary<Section> Mechanical Ventilation<Level> Application/Analysis

49. A 36-year-old woman with a long history of asthma is admitted to the critical care unit from the emergency department. Blood pressure is 110/80 mm Hg, heart rate is 110 beats/min, and respiratory rate is 28 breaths/min. Arterial blood gas results on 24% oxygen are as follows: pH 7.48 PaCO2 29 mm Hg PaO2 59 mm Hg HCO3 26 mEq/L Because she already has been taking inhaled sympathomimetic bronchodilators and steroids but is still having severe bronchospasm, an aminophylline infusion is initiated after a loading dose. Which of the following is associated with decreased aminophylline metabolism? a. Smokersb. Renal insufficiency c. Heart failured. Obesity

Correct answer: cRationale: Aminophylline is a xanthine bronchodilator and recently has been shown to have an antiinflammatory effect. Aminophylline frequently is used for asthma and other bronchospastic processes. Aminophylline is metabolized by the liver to inactive metabolites, so renal failure does not increase the risk of aminophylline toxicity. Heart failure is associated with hepatic enlargement, which decreases the efficiency of the liver in metabolizing toxins and drugs. Smokers increase metabolism of aminophylline and require a higher dosage. Obesity and renal insufficiency do not affect aminophylline clearance. Test-Taking Strategy: Read the question carefully. You probably were tempted to say renal insufficiency, but the question is about metabolism. The liver usually is the organ that metabolizes or biotransforms a drug into active and inactive metabolites, and then the kidney excretes the metabolites, although sometimes the drug is excreted unchanged by the kidney. Choose option c because the question asks about metabolism (not elimination) and heart failure affects the liver.<System> Pulmonary<Section> Asthma; Pharmacology<Level> Knowledge/Comprehension

17. A 56-year-old man was admitted to the intensive care unit with a diagnosis of status asthmaticus. Asthma is best characterized by which of the following?a. Alveolar destructionb. Alveoli filled with exudate c. Bronchiolar swelling and spasmd. Loss of supporting fibers for the bronchiolar walls

Correct answer: cRationale: Asthma is characterized by bronchiolar narrowing caused by mucosal swelling and spasm along with the production of thick, tenacious mucus. Remember asthma is reactive airway disease.Test-Taking Strategy: Because asthma is an obstructive disease, it affects the airways. Options a and b concern the alveoli rather than the airways, so eliminate them. Option d is loss of supportive fibers, which implies irreversibility. Because asthma is a reversible obstructive disease, option c is the most logical choice.<System> Pulmonary<Section> Asthma<Level> Knowledge/Comprehension

31. A 52-year-old man had surgery yesterday for a ruptured abdominal aortic aneurysm. He has a history of hypertension and coronary artery disease but no history of pulmonary problems. He is intubated and mechanically ventilated during the night after surgery. The next morning the patient is on intermittent mandatory ventilation (IMV) with a rate of 10 breaths/min, tidal volume of 650 mL, fraction of inspired oxygen of 0.4, and positive end-expiratory pressure of 5 cm H2O. Vital signs are stable, and his total respiratory rate is 16 breaths/min. Arterial blood gases are pH, 7.39; PaCO2, 42 mm Hg; HCO3, 22 mEq/L; and PaO2, 96 mm Hg. In preparation for weaning, spontaneous ventilatory parameters are measured. Spontaneous tidal volume is 400 mL, vital capacity is 950 mL, minute ventilation is 9 L/min, respiratory rate is 22 breaths/min, and maximal inspiratory pressure is −29 cm H2O. The patient weighs 69 kg. After 30 minutes on pressure support with 40% oxygen, the patient has the following spontaneous ventilatory parameters and arterial blood gases: Spontaneous Ventilatory Parameters Tidal volume, 425 mLVital capacity, 900 mLMaximal inspiratory pressure, −40 cm H2OMinute ventilation, 8.5 L/min Respiratory rate, 20 breaths/min Arterial Blood GasespH, 7.38PaCO2, 43 mm Hg HCO3, 22 mEq/L PaO2, 98 mm Hg Arterial oxygen saturation, 98% What should be the next step in the weaning process?a. Put the patient back on IMV.b. Administer bronchodilator nebulizer treatment.c. Extubate the patient and administer 40% oxygen via mask.d. Continue trial for at least 1 hour more to see whether the patient will improve further.

Correct answer: cRationale: Based on these ventilatory parameters and arterial blood gases after the short- term breathing trial, he should be extubated. Remember that leaving the patient intubated without mechanical ventilator support greatly increases airway resistance and work of breathing. Do not delay extubation.Test-Taking Strategy: Eliminate the extremes, options a and d. Option b can be eliminated because there is no indication of bronchospasm and wheezing. Choose option c. <System> Pulmonary<Section> Mechanical Ventilation<Level> Synthesis/Evaluation

32. A patient had a nephrectomy yesterday. She is being monitored closely for indications of hypoventilation and atelectasis. Which characteristic best defines hypoventilation?a. Respiratory rate less than 12 breaths/min b. pH less than 7.35c. PaCO2 greater than 45 mm Hg d. PaO2 less than 60 mm Hg

Correct answer: cRationale: Because minute ventilation is affected by respiratory rate and tidal volume, respiratory rate alone is not definitive of hypoventilation. Also, a factor in this issue is the unknown of carbon dioxide production based on metabolic rate (e.g., greater ventilation is required when hypermetabolism exists because carbon dioxide production then is increased). A pH of less than 7.35 could result from metabolic acidosis or from respiratory acidosis, so it is not definitive. PaO2 tells you about oxygenation and not specifically ventilation. Patients can be hypoxemic for reasons other than hypoventilation. The most specific answer is PaCO2. If PaCO2 is greater than 45 mm Hg, hypoventilation exists. If PaCO2 is less than 45 mm Hg, hyperventilation exists.Test-Taking Strategy: Respiratory rate is only half of ventilation because it does not tell you anything about depth. Eliminate option a. Respiratory and metabolic status affects pH, so pH is not specific to ventilation status. Eliminate option b. Remember that PaCO2 is reflective of ventilation, whereas PaO2 is reflective of oxygenation. Eliminate option d and choose option c.<System> Pulmonary<Section> Pulmonary Assessment<Level> Knowledge/Comprehension

54. A 52-year-old, 70-kg man has just arrived in the critical care unit after exploratory thoracotomy following thoracic injuries in a motor vehicle collision. He had massive hemorrhage and has had four units of packed red blood cells. His current hemoglobin is 7.8 g/dL. He is scheduled to receive more blood as soon as it is available. His pleural chest tube is functioning well and draining about 50 to 100 mL/hr. He is on a mechanical ventilator on assist-control mode with a tidal volume of 700 mL and fraction of inspired oxygen of 0.4. Which of the following would not be an indication of hypoxemia in this patient? a. Restlessnessb. Decrease in arterial oxygen saturation (SaO2) by pulse oximetry c. Cyanosisd. Decrease in PaO2 by arterial blood gases

Correct answer: cRationale: Central cyanosis occurs when 5 g of hemoglobin is desaturated. This would be a very late, if not impossible, sign in a patient this anemic. Restlessness would indicate cerebral hypoxia. A decrease in SaO2 and/or PaO2 would indicate a decrease in the amount of oxygen attached to the hemoglobin and dissolved in the blood, respectively.Test-Taking Strategy: This is a negatively stated question, so read slowly and carefully. For which should you not monitor? The answer is the one that is not possible in an anemic patient. Choose cyanosis, option c.<System> Pulmonary<Section> Thoracic Trauma<Level> Synthesis/Evaluation

29. The high-pressure alarm on a volume ventilator sounds. This would not be caused by which of the following?a. Biting on the tubeb. Pneumothorax c. Cuff leakd. Excessive secretions

Correct answer: cRationale: Cuff leak would cause the low exhaled volume alarm to sound. Biting on the tube, excessive secretions, and pneumothorax would increase the resistance to airflow into the lungs and cause the high-pressure alarm to sound.Test-Taking Strategy: Read the question carefully and note the word not. Missing this one word changes the entire interpretation of the question. To identify the exception, think of the patient-ventilator circuit as a closed circuit. One of the options listed would open the circuit. Choose option c.<System> Pulmonary<Section> Airway; Mechanical Ventilation<Level> Knowledge/Comprehension

15. A 56-year-old man was admitted to the intensive care unit with a diagnosis of status asthmaticus. What are the three most common causes of status asthmaticus?a. Exposure to allergens, experiencing a wide variety of strong emotions, not allowing adequate rest periods in the daytime b. Noncompliance with medication regimen, inadequate rest periods during the waking hours, the gradual ineffectiveness of medicationsc. Exposure to allergens, noncompliance with medication regimen, respiratory infection d. Pulmonary trauma, gradual ineffectiveness of medications, experiencing a wide variety of strong emotions

Correct answer: cRationale: Exposure to allergens, noncompliance with prescribed therapy, and respiratory infection are the three most common causes of status asthmaticus.Test-Taking Strategy: In questions like this with several answers in each option, use a true-false approach. All of the answers in the option must be correct for that option to be the correct answer. Go with what you know. Allergens are in options a and c. The option a component "not allowing adequate rest periods in the daytime" does not seem logical. In addition, status asthmaticus is an acute event, so rule out gradual causes such as gradual ineffectiveness of medications as in options b and d. Go with option c.<System> Pulmonary<Section> Asthma<Level> Knowledge/Comprehension

42. A patient is admitted to the critical care unit with multiple trauma resulting from a motor vehicle collision. She is becoming increasingly dyspneic. Which of the following findings would be consistent with a diagnosis of massive hemothorax?a. Hyperresonance to percussion, clear breath sounds on affected side, and tracheal deviation toward the affected side b. Resonance to percussion, absent breath sounds on affected side, and tracheal deviation toward the unaffected sidec. Dullness to percussion, absent breath sounds on affected side, and tracheal deviation toward unaffected side d. Tympany to percussion, absent breath sounds on affected side, and trachea midline

Correct answer: cRationale: Fluid, such as blood, will be dull to flat to percussion. The blood in the pleural space compresses normal lung tissue and causes diminished to absent breath sounds. A large volume of blood in the pleural space may cause the trachea to shift away from the affected side toward the unaffected side.Test-Taking Strategy: Because hemothorax is not normal, eliminate the normal findings: clear breath sounds (a), resonance to percussion (b), and trachea midline (d). Choose option c.<System> Pulmonary<Section> Thoracic Trauma<Level> Application/Analysis

43. A patient is admitted to the critical care unit with multiple trauma resulting from a motor vehicle collision. She is becoming increasingly dyspneic. Chest x-ray film confirms massive hemothorax. A chest tube with water-seal drainage is initiated. The patient is typed and crossmatched for blood, and despite prompt surgical intervention, the patient still requires 10 units of banked blood. What effect would the infusion of this volume of banked blood have on tissue oxygenation? a. Hemoglobin more readily picking up oxygen at the lungb. Hemoglobin more readily releasing oxygen into the tissues c. Hemoglobin less readily releasing oxygen into the tissues d. Hemoglobin less readily picking up oxygen at the lung

Correct answer: cRationale: Large volumes of banked blood cause a shift of the oxyhemoglobin dissociation curve to the left because banked blood is low in 2,3-diphosphoglycerate (2,3-DPG). This is a by-product of glucose metabolism on the hemoglobin molecule that encourages the dissociation of oxygen from hemoglobin. A shift of the curve to the left increases the affinity between hemoglobin and oxygen and results in hemoglobin less readily releasing oxygen at the tissue level.Test-Taking Strategy: Large volumes of banked blood are associated with decreased levels of 2,3-DPG. Alkalosis, hypothermia, and decreased levels of 2,3-DPG cause a shift of the oxyhemoglobin dissociation curve to the left. Remember: Left causes great pickup (at the lung) but poor drop-off (at the tissue).<System> Pulmonary<Section> Thoracic Trauma<Level> Synthesis/Evaluation

67. A 55-year-old woman is admitted with multiple trauma after a motor vehicle collision. She sustained multiple fractures, including to her right femur. The next evening she is dyspneic and confused and has petechiae over her chest, upper arms, and face. These clinical manifestations are likely to indicate which of the following?a. Bacterial endocarditisb. Disseminated intravascular coagulation (DIC)c. Fat embolusd. Hemorrhagic shock

Correct answer: cRationale: Long-bone fracture predisposes the patient to fat embolus. These are classic manifestations of fat embolus: dyspnea, confusion, and petechiae.Test-Taking Strategy: Petechiae would be present in bacterial endocarditis and DIC, but these conditions are unlikely to be associated directly with dyspnea and confusion. Hemorrhagic shock would not cause any of these signs and symptoms directly. Associate the manifestations with the multiple trauma and femur fracture. Choose option c. <System> Pulmonary<Section> Pulmonary Embolism<Level> Synthesis/Evaluation

41. Which of the following describes the pathophysiologic effect of acute respiratory distress syndrome (ARDS) on the lung?a. Decreased pulmonary vascular resistanceb. Increased functional residual capacity c. Decreased pulmonary compliance d. Decreased alveolar surface tension

Correct answer: cRationale: Lungs become stiff as a result of alveolar collapse and surfactant deficiency. This decreased compliance (expandability) causes an increase in the work of breathing and patient fatigue. Pulmonary vascular resistance is increased, functional residual capacity is decreased, and alveolar surface tension is increased.Test-Taking Strategy: Remember that the pressure gauge on the mechanical ventilator reflects changes in compliance. Patients with ARDS usually have high peak inspiratory pressures indicative of decreased pulmonary compliance. Choose option c.<System> Pulmonary<Section> Acute Respiratory Distress Syndrome<Level> Knowledge/Comprehension

71. Which is a normal oxygen consumption index (VO2I)? a. Approximately 50 mL/min/m2b. Approximately 100 mL/min/m2c. Approximately 150 mL/min/m2 d. Approximately 200 mL/min/m2

Correct answer: cRationale: Normal arterial oxygen saturation is ~100%, and normal venous oxygen saturation is ~75%. The tissues used 25%. The normal arterial oxygen content is ~20 mL/dL, and normal venous oxygen content is ~15 mL/dL. The tissues used 25%. The normal oxygen delivery (DO2) is ~1000 mL/min, and normal VO2 is ~250 mL/min. The tissues used 25%. The normal DO2I is ~600 mL/min/m2, so consider what is 25% of 600 mL/min/m2? The normal VO2I is ~150 mL/min/m2.Test-Taking Strategy: Remember those relationships. Normal parameters are 100% out, 25% use, and 75% reserve.<System> Pulmonary<Section> Hypoxia<Level> Knowledge/Comprehension

70. A 28-year-old woman is admitted to the critical care unit from the emergency department with a diagnosis of asthma. Her initial arterial blood gases on a 28% Venturi mask are as follows: pH 7.48 PaCO2 30 mm Hg HCO3 24 mEq/L PaO2 64 mm Hg Which of the following repeat arterial blood gases on 40% oxygen indicate that the patient's condition is worsening?a. pH 7.48, PaCO2 30 mm Hg, PaO2 68 mm Hgb. pH 7.46, PaCO2 32 mm Hg, PaO2 61 mm Hg c. pH 7.40, PaCO2 40 mm Hg, PaO2 62 mm Hg d. pH 7.39, PaCO2 30 mm Hg, PaO2 60 mm Hg

Correct answer: cRationale: The case study shows stage II asthma. Option c shows stage III asthma. The patient is still breathing at a fast rate, but carbon dioxide is starting to be retained as evidenced by the increase of the PaCO2 into normal range. Options a and b are still stage II. Option d shows a respiratory alkalosis with a metabolic acidosis because you would have expected the pH to be in an alkalotic range with the PaCO2 of 30.Test-Taking Strategy: The only arterial blood gases that would indicate a higher PaCO2 is option c. Remember that PaCO2 reflects adequacy of alveolar ventilation. Choose option c. <System> Pulmonary<Section> Asthma<Level> Synthesis/Evaluation

11. A 24-year-old man is admitted to the critical care unit after sustaining a pulmonary contusion in a motor vehicle collision. He has no history of cardiac or pulmonary disease. Over the last few hours he has been complaining of increasing dyspnea, his respiratory rate has been increasing, and his oxygen saturation via pulse oximetry has been decreasing. Breath sound assessment reveals fine crackles bilaterally. Arterial blood gases reveal respiratory alkalosis and hypoxemia. Chest x-ray film reveals patchy infiltrates. Acute respiratory distress syndrome (ARDS) is diagnosed. Oxygen therapy is initiated, and arterial blood gases are monitored closely. Ventilator settings are initially tidal volume of 700 mL, rate of 12 breaths/min, and fraction of inspired oxygen (FiO2) of 1.0, with positive end-expiratory pressure (PEEP) at 5 cm H2O. The PEEP is gradually increased to 20 cm H2O so that the FiO2 can be reduced while maintaining a PaO2 greater than 60 mm Hg. Suddenly the high-pressure alarm on the ventilator sounds. The patient indicates shortness of breath. His blood pressure is decreased from the last hour, and the heart rate is increased. Breath sounds are now decreased on the left side. Which of the following is most likely to be the problem? a. Pulmonary embolismb. Cardiac tamponadec. Pneumothoraxd. Worsening of acute respiratory distress syndrome (ARDS)

Correct answer: cRationale: The increasing levels of PEEP predispose the patient to barotrauma. The high- pressure alarm indicates a decrease in compliance (or airway resistance), and the unequal breath sounds help to pinpoint pneumothorax as the cause of these changes.Test-Taking Strategy: Cardiac tamponade (option b) would not affect breath sounds. Worsening of ARDS (option d) would cause a gradual decrease in compliance and an increase in peak inspiratory pressure. It also probably would be a bilateral change. Pulmonary embolus (option a) would increase vascular pressures (e.g., pulmonary artery pressure and pulmonary vascular resistance) but would not affect lung inflation pressures. Pneumothorax (option c) is the only choice that would cause a sudden change in lung compliance and a unilateral change in breath sounds.<System> Pulmonary<Section> Pneumothorax; Mechanical Ventilation<Level> Synthesis/Evaluation

28. A 54-year-old man has just returned to the critical care unit from the postanesthesia care unit. He has a 60-pack-year history of cigarette smoking and had a right lower lobectomy performed earlier today for treatment of lung cancer. He is still intubated and on a positive pressure mechanical ventilator. The nurse notes diminished breath sounds in the left posterior base. What would these findings most likely indicate? a. Pleurisyb. Pneumoniac. Atelectasisd. Presence of chest tube

Correct answer: cRationale: The most likely cause is atelectasis. Deep breathing and incentive spirometry are indicated to prevent pneumonia from developing.Test-Taking Strategy: Diminished breath sounds indicate diminished air flow. Atelectasis is the logical choice. Pleurisy causes a pleural friction rub. Pneumonia causes bronchial breath sounds. The chest tube does not cause any specific breath sound, though there may be gurgling audible if the suction is not clamped during auscultation. Choose option c. <System> Pulmonary<Section> Thoracotomy; Pulmonary Assessment<Level> Application/Analysis

20. A 42-year-old man is admitted to the critical care unit with smoke inhalation and acute respiratory distress syndrome (ARDS). He is intubated, and the following mechanical ventilation is initiated: fraction of inspired oxygen, 0.6; intermittent mandatory ventilation, 10 breaths/min; tidal volume, 650 mL; positive end-expiratory pressure, 15 cm H2O. Arterial blood gases are pH, 7.39; PaCO2, 42 mm Hg; HCO3, 24 mEq/L; and PaO2, 70 mm Hg. Massive atelectasis occurs in acute respiratory distress syndrome. Which of the following are the two major causes of this alveolar collapse? a. Increased pulmonary vascular resistance and increased pulmonary compliance b. Increased pulmonary compliance and pulmonary edema c. Surfactant deficiency and pulmonary edemad. Mucous plugs and bronchospasm

Correct answer: cRationale: The role of surfactant is to keep the alveoli open at low distending volumes (e.g., at the end of expiration). Surfactant deficiency occurs in ARDS as a result of damage to the type II pneumocytes that make surfactant. These cells are sensitive to oxygenation changes, so shock states that decrease the delivery of oxygen to the lung also cause damage to the type II pneumocytes. Interstitial edema and eventually intraalveolar edema occur as a result of the increase in the permeability of the pulmonary capillary. This intraalveolar edema also dilutes and inactivates surfactant.Test-Taking Strategy: Go with what you know. Lungs get stiff in ARDS. This means that pulmonary compliance decreases, so eliminate options a and b. Eliminate option d because edema, not mucus, is the problem in ARDS. Surfactant deficiency caused by damage to the type II pneumocytes decreases pulmonary compliance and increases surface tension. Increased capillary permeability and noncardiac pulmonary edema with resultant refractory hypoxemia are the hallmarks of ARDS. Choose option c.<System> Pulmonary<Section> Acute Respiratory Distress Syndrome<Level> Knowledge/Comprehension

46. Before transferring a patient with a total hip replacement and history of heart failure to a medical-surgical unit, the nurse conducts a pretransfer physical assessment. Suddenly the patient develops tachycardia, tachypnea, dyspnea, and generalized chest discomfort. Which of the following is the most likely cause of these symptoms?a. Myocardial infarctionb. Tension pneumothoraxc. Pulmonary embolismd. Acute anxiety response

Correct answer: cRationale: The sudden onset of tachycardia, tachypnea, dyspnea, and chest discomfort is most likely to be caused by pulmonary embolism in this patient. Although acute anxiety response is not unusual at transfer time, paresthesia is more likely than chest discomfort in anxiety attacks. Tension pneumothorax is most likely to be associated with chest trauma or mechanical ventilation. Myocardial infarction is possible but less likely in this patient. Test-Taking Strategy: The sudden severe change with the symptoms identified in this case study should make you suspect that option c is the correct answer.<System> Pulmonary<Section> Pulmonary Embolism<Level> Application/Analysis

48. A 36-year-old woman with a long history of asthma is admitted to the critical care unit from the emergency department. Blood pressure is 110/80 mm Hg, heart rate is 110 beats/min, and respiratory rate is 28 breaths/min. Arterial blood gas results on 24% oxygen are as follows: pH 7.48 PaCO2 29 mm Hg PaO2 59 mm Hg HCO3 26 mEq/L The patient tells the nurse that she took her inhaled bronchodilator before coming to the hospital. She says that she uses albuterol (Proventil). Which of the following describes the properties of sympathomimetics used as bronchodilators (such as albuterol)?.a. Alpha stimulating activity and therefore bronchodilating properties b. Beta1 stimulating activity and therefore bronchodilating propertiesc. Beta2 stimulating activity and therefore bronchodilating propertiesd. Dopaminergic stimulating activity and therefore bronchodilating properties

Correct answer: cRationale: This question is about receptors of the sympathetic nervous system. Alpha receptors cause vasoconstriction when stimulated. Beta1 receptors cause an increase in heart rate, contractility, and conductivity when stimulated. Beta2 receptors cause bronchodilation when stimulated. Dopaminergic receptors cause vasodilation, primarily of the renal and mesenteric circulation, when stimulated. The advantage of albuterol (Proventil), terbutaline (Brethine), metaproterenol (Alupent), and isoetharine (Bronkosol) is that they are more respiratory selective than epinephrine and isoproterenol. They stimulate beta2 more than beta1, thereby causing less tachycardia and nervousness.Test-Taking Strategy: Remember: beta1 equals one heart; beta2 equals two lungs. Stimulation of beta2 receptors causes bronchodilation.<System> Pulmonary<Section> Asthma; Pharmacology<Level> Knowledge/Comprehension

40. A 57-year-old man with acute respiratory failure is receiving enteral nutrition. Which of the following nursing interventions would reduce the risk of aspiration in patients receiving enteral feedings?a. Keep the head of the bed flat rather than elevated. b. Use intermittent bolus feedings rather than continuous feedings.c. Use small-bore feeding tubes rather than large-bore nasogastric tubes. d. Use gastric tubes rather than jejunostomy tubes.

Correct answer: cRationale: To prevent aspiration in patients being nourished enterally, the head of the bed should be elevated. Continuous feedings cause less gastric distention than intermittent feedings. Small feeding tubes cause less gastroesophageal sphincter incompetence that do larger-gauge nasogastric or Levine tubes. Jejunostomy tubes are less likely to be associated with aspiration than gastric tubes because they are below the gastroesophageal sphincter and the pyloric sphincter. Remember that feedings below the stomach must be continuous rather than intermittent because there is no storage capacity.Test-Taking Strategy: Consider which technique would cause the least chance of a bolus of feeding coming back into the pharynx to be aspirated into the lungs. Open sphincters, large boluses, and a pharynx level with the stomach would increase the chance of aspiration. Choose option c.<System> Pulmonary<Section> Aspiration<Level> Application/Analysis

14. A 70-kg male patient with acute respiratory failure is being weaned from mechanical ventilation using the intermittent mandatory ventilation (IMV) method. Ventilator settings and current arterial blood gas results are as follows: Ventilator Settings Fraction of inspired oxygen, 0.35 Tidal volume, 700 mLIMV, 4 breaths/minute Arterial Blood Gases pH, 7.29HCO3, 22 mEq/L PaCO2, 52 mm Hg PaO2, 96 mm Hg While performing assessment of the patient, the nurse asks if his breathing is okay. He answers "yes" in an audible tone. Appropriate action would be to:a. add air to the cuff until the patient can no longer speak audibly.b. order a chest x-ray film to check endotracheal tube placement. c. remove all air from the cuff and reinflate it to minimal occlusive volume. d. notify the doctor immediately.

Correct answer: cRationale: Tube placement is less likely to be the problem than cuff leak. Adding air regardless of the amount of air already in the cuff could certainly cause excessive tracheal pressure and potential necrosis. The best option is to reestablish the appropriate volume of air in the cuff using minimal occlusive volume or minimal leak volume. Measuring the cuff pressure with a cuff pressure gauge is also indicated.Test-Taking Strategy: Although option a certainly may solve the problem (a cuff leak), it also may create another problem (tracheal necrosis). Remember that ordering a chest x-ray film (option b) involves expenditure of money (resources) and should be done only if necessary. Notifying the doctor (option d) is reserved for those situations where no additional assessment is indicated and there is nothing definitive the nurse can do. Option c is the safest and most appropriate first action to take.<System> Pulmonary<Section> Airway<Level> Application/Analysis

56. A patient receiving an epidural narcotic is admitted to the critical care unit. Which of these findings requires immediate attention?a. Urinary retentionb. Itching c. Respiratory rate of 8 breaths/min d. Heart rate of 110 beats/min

Correct answer: cRationale: Urinary retention and itching are common adverse effects of epidural analgesia. They frequently are treated with small doses of naloxone (Narcan). Bradycardia is a more common adverse effect of epidural opiates than is tachycardia. The most serious adverse effect of excessive opiate is ventilatory depression. A respiratory rate of 8 breaths/min requires immediate attention.Test-Taking Strategy: Consider which of these options has the most serious consequences. Choose option c. In addition, if you are not sure, use the ABC approach. Airway and breathing are always a first priority.<System> Pulmonary<Section> Acute Respiratory Failure<Level> Synthesis/Evaluation

66. A patient with flail chest is admitted to the critical care unit. He is given vecuronium (Norcuron) and put on control mode positive pressure ventilation. What other medication must be given?a. Propranolol (Inderal) b. Diazepam (Valium) c. Morphine sulfated. Lorazepam (Ativan)

Correct answer: cRationale: Vecuronium is a nondepolarizing muscle paralytic. Vecuronium does not affect consciousness or comfort. Although in some patients diazepam or lorazepam may be adequate for sedation, this patient is expected to have a great deal of pain from the rib or sternal fractures. Morphine will provide this analgesia. Propranolol (Inderal) sometimes is given with pancuronium, another nondepolarizing muscle paralytic, because severe tachycardia frequently is caused by pancuronium.Test-Taking Strategy: Consider a patient with multiple rib fractures on mechanical ventilation. Analgesics would be needed with or without the muscle paralytic agents. Choose option c.<System> Pulmonary<Section> Thoracic Trauma; Pharmacology<Level> Synthesis/Evaluation

A patient is admitted to the critical care unit with acute exacerbation of COPD caused by respiratory infection. Temperature is 39° C, blood pressure is 150/82 mm Hg, heart rate is 110 beats/min, and respiratory rate is 24 breaths/min. Chest x-ray film confirms that there is left upper lobe pneumonia. Arterial blood gases reveal the following: pH 7.29 PaCO2 62 mm Hg HCO3 30 mEq/L PaO2 52 mm Hg The acidosis and a fever in this patient would result in which of the following changes in the oxyhemoglobin dissociation curve? a. A shift to the right and increased saturation b. A shift to the left and increased saturation c. A shift to the left and decreased saturation d. A shift to the right and decreased saturation

Correct answer: d Rationale: Acidosis and hyperthermia shift the oxyhemoglobin dissociation curve to the right. This decreases affinity between hemoglobin and oxygen and results in a lower oxygen saturation for a given PaO2. The shift of the curve to the right, decreasing affinity, means it is more difficult for the hemoglobin to pick up oxygen at the lung, but it gives up oxygen easily at the tissue. In addition, remember that with a normal midline curve, a PaO2 of 60 mm Hg correlates with an SaO2 of 90%. Test-Taking Strategy: Remember the following: Rise In 2,3-DPG H+ (acidosis) Temperature (hyperthermia) to help you remember which factors shift the curve right and the opposites to the left. The issue here is affinity. A shift to the right decreases affinity, and a shift to the left increases affinity. This is why the saturation is lower for a given PaO2 when the curve is shifted to the right. Acidosis, hyperthermia, and increased 2,3-DPG (2,3-diphophoglycerate) shift the curve to the right. Envision the curve with oxygen saturation on the vertical axis and PaO2 on the horizontal axis. A shift right would result in a lower saturation for a given PaO2, whereas a shift left would result in a higher saturation for a given PaO2. <System> Pulmonary <Section> Acute Respiratory Failure

97. Which of the following surgical procedures would be associated with the lowest incidence of postoperative pulmonary complications?a. Flankb. Thoracic c. Upper abdominal d. Lower abdominal

Correct answer: d Rationale: Flank, thoracic, and upper abdominal incisions are associated with the highest risk of pulmonary complications because the patient experiences pain with each breath. Without adequate analgesia, this leads to splinting, hypoventilation, and atelectasis. Test-Taking Strategy: Choose the surgery farthest away from the thorax.<System> Pulmonary<Section> Acute Respiratory Failure<Level> Knowledge/Comprehension

86. Which of the following does not shift the oxyhemoglobin dissociation curve to the left or the right?a. Blood pHb. 2,3-diphosphoglycerate (2,3-DPG) levels c. Body temperature d. Cardiac output

Correct answer: dRationale: Body temperature, body pH, PaCO2 levels, and 2,3-DPG levels affect the oxyhemoglobin dissociation curve. Cardiac output does not directly affect the curve. Test-Taking Strategy: None<System> Pulmonary<Section>Oxygen<Level> Knowledge/Comprehension

19. A 56-year-old man was admitted to the intensive care unit with a diagnosis of status asthmaticus. Drug therapy most likely to be prescribed for a patient with status asthmaticus will include which of the following drug categories?a. Bronchodilators and anticoagulants b. Corticosteroids and diureticsc. Antibiotics and diureticsd. Bronchodilators and corticosteroids

Correct answer: dRationale: Bronchodilators improve airflow and decrease airway resistance, and corticosteroids decrease the inflammatory process. Antibiotics are useful only if bacterial infection is present. Diuretics would be detrimental because this patient may be extremely dehydrated. Anticoagulants would not be indicated.Test-Taking Strategy: Go with what you know. Because the pathophysiology includes bronchospasm, bronchodilators certainly would be indicated. Options a and d include bronchodilators. Option a includes anticoagulants, which would not be helpful with asthma, so choice option d.<System> Pulmonary<Section> Asthma; Pharmacology<Level> Knowledge/Comprehension

27. A 54-year-old man has just returned to the critical care unit from the postanesthesia care unit. He has a 60-pack-year history of cigarette smoking and had a right lower lobectomy performed earlier today for treatment of lung cancer. He is still intubated and on a positive pressure mechanical ventilator. Which of the following descriptions is indicative of a pleural air leak? a. No fluctuation of the water level in the water-seal chamber with ventilation b. Fluctuation of the water level in the water-seal chamber with ventilationc. Bubbling in the suction control chamberd. Bubbling in the water-seal chamber

Correct answer: dRationale: Bubbling in the water-seal chamber indicates that there is still air in the pleural space that is being allowed to escape safely through a water seal.Test-Taking Strategy: First eliminate normal functioning: options b and c. Option a indicates a reexpanded lung or an occlusion in the circuit. Choose option d.<System> Pulmonary<Section> Chest Tube<Level> Application/Analysis

75. Which of the following would not be an indication that the intubated and mechanically ventilated patient should be suctioned?a. Rhonchi audible throughout lung fieldsb. Coughing during the inspiratory cycle of the ventilator c. Visible secretions in the tracheostomy or endotracheal tube d. Fine crackles audible in the lung bases

Correct answer: dRationale: Crackles indicate an intraalveolar process. The alveoli cannot be suctioned. Rhonchi indicate fluid or mucus in the airway and indicate that a patient should be helped to cough or that secretions should be suctioned. Coughing during the inspiratory cycle of the ventilator or the high-pressure alarm of the ventilator sounding indicates that the patient requires suctioning of secretions. Visible secretions in the tracheostomy or endotracheal tube indicate that the patient requires suctioning of secretions. Patients should not receive suctioning routinely.Test-Taking Strategy: Options a, b, and c indicate that there is mucus. Crackles indicate intraalveolar fluid, atelectasis, or pulmonary fibrosis, which are not amenable to suctioning. Remember that this is a negatively stated question, so read slowly and carefully. Choose option d as the one that does not indicate that suctioning is needed.<System> Pulmonary<Section> Airway<Level> Knowledge/Comprehension

73. A 65-year-old woman reports severe dyspnea 2 days after abdominal surgery. She is transferred to the critical care unit. On 5 L of oxygen by nasal cannula, her arterial blood gases are as follows: pH- 7.39 PaCO2- 35 mm Hg HCO3- 19 mEq/L PaO2- 40 mm Hg Arterial oxygen saturation- 75% Diagnosis of pulmonary embolism is made by pulmonary arteriography. The patient is now receiving 100% oxygen via a non-rebreathing mask. Which of the following would not be an indication for aggressive therapy with fibrinolytic agents for this patient?a. Refractory hypoxemia b. Obstruction of flow to a lung lobe or multiple lung segments c. Right ventricular failured. Pleural friction rub

Correct answer: dRationale: Fibrinolytic agents are indicated for massive and submassive pulmonary embolism. Unfortunately, patients with massive pulmonary embolism may not survive long enough for treatment with fibrinolytic agents. Hemodynamic consequences, refractory hypoxemia, and significant obstruction by arteriogram are the usual clinical indications for fibrinolytic agents. Pleural friction rub is an indication of pulmonary infarction and occurs 24 to 72 hours after the pulmonary embolism. This would be too late to reverse the hypoperfusion injury to the lung.Test-Taking Strategy: Remember that this is a negatively stated question, so read slowly and carefully. Option d is different for two reasons. The first reason is that it is later than the other three options. The second reason is that it is less life threatening. Choose option d. <System> Pulmonary<Section> Pulmonary Embolism<Level> Application/Analysis

23. A 54-year-old man has just returned to the critical care unit from the postanesthesia care unit. He has a 60-pack-year history of cigarette smoking and had a right lower lobectomy performed earlier today for treatment of lung cancer. He is still intubated and on a positive pressure mechanical ventilator. When assessing his chest tube, the nurse notes fluctuation in the water-seal chamber. The water falls during inspiration and rises during expiration. This indicates which of the following? a. Tension pneumothoraxb. Excessive suctionc. A pleural air leakd. Normal pleural pressure changes

Correct answer: dRationale: Fluctuation in the water-seal chamber is expected because it communicates the changes in intrapleural pressure. The changes described are normal for a patient on a positive pressure volume mechanical ventilator. They would be the opposite if the patient were breathing spontaneously.Test-Taking Strategy: Pleural air leak causes persistent bubbling in the water-seal chamber, so eliminate option c. Excessive suction would cause excessive bubbling in the suction control chamber, so eliminate option b. Tension pneumothorax may result if the chest tube is not functioning (not fluctuating), so eliminate option a. Note: Identifying a condition as normal is frequently challenging. If you can eliminate the abnormal conditions, the scenario likely is normal.<System> Pulmonary<Section> Chest Tubes; Thoracotomy<Level> Synthesis/Evaluation

93. A patient is admitted after a motor vehicle collision which resulted in his chest impacting the steering wheel. Which of the following assessment methods would be used to detect that flail chest has occurred as the result of this impact?a. Percuss the chest for hyperresonance.b. Auscultate the breath sounds for crackles.c. Palpate the chest for crepitus.d. Inspect the chest for paradoxical chest wall movement.

Correct answer: dRationale: Hyperresonance to percussion indicates pneumothorax. Crackles indicate atelectasis or pulmonary edema. Crepitus may be felt over bone fractures, but it is not specific enough to identify a flail chest. Paradoxical chest wall movement would be the option most specific to flail chest.Test-Taking Strategy: Always look for the most specific (best) answer.<System> Pulmonary<Section> Thoracic Trauma<Level> Application/Analysis

81. Which of the following is the best indicator of tissue oxygenation? a. PaO2b. Arterial oxygen saturation (SaO2)c. Arterial oxygen content (CaO2) d. Oxygen delivery (DO2)

Correct answer: dRationale: Ninety-seven percent of oxygen is attached to the hemoglobin (SaO2). The other 3% is dissolved in the plasma (PaO2). So SaO2 is a better indication of tissue oxygenation than is PaO2. The content of oxygen in the arterial blood is called CaO2. This is calculated as oxygen capacity multiplied by SaO2. One calculates oxygen capacity by multiplying the hemoglobin (in grams) by 1.34. Because hemoglobin is required to carry the oxygen, CaO2 is more encompassing than SaO2 alone because CaO2 incorporates SaO2 and hemoglobin. Oxygen delivery would add cardiac output to the picture. DO2 = Hgb × 1.34 × SaO2 × CO × 10. Normal DO2 is ~1000 mL/min.Test-Taking Strategy: Look for the option that includes as many of the components of tissue oxygenation as possible. They actually are in order of significance. PaO2 reflects only 3% of oxygen in the blood (oxygen that is dissolved in the plasma). SaO2 reflects the saturation of the hemoglobin. CaO2 reflects the amount of hemoglobin and its degree of saturation. DO2 reflects the amount of hemoglobin and its degree of saturation and the rate at which blood is being delivered to the tissues. Choose option d.<System> Pulmonary<Section> Hypoxia<Level> Application/Analysis

50. Which of the following prevents pulmonary emboli in a patient with recurrent deep vein thrombosis?a. Heparinb. Warfarin (Coumadin) c. Antiembolic stockings d. Greenfield filter

Correct answer: dRationale: Only the Greenfield filter will prevent pulmonary emboli from recurrent deep vein thrombosis. The other options may prevent the thrombi from developing but will not prevent them from getting to the lung. A Greenfield filter, or vena cava umbrella, is placed when the patient has recurrent pulmonary embolism despite anticoagulant therapy or when anticoagulants are contraindicated.Test-Taking Strategy: Read the question carefully. It says that there are recurrent deep vein thromboses. Now, how do you keep them from getting to the lung? Differentiate between prevention of thrombosis versus prevention of pulmonary embolism in a patient with thrombosis.<System> Pulmonary<Section> Pulmonary Embolism; Pharmacology<Level> Synthesis/Evaluation

21. A 42-year-old man is admitted to the critical care unit with smoke inhalation and acute respiratory distress syndrome (ARDS). He is intubated, and the following mechanical ventilation is initiated: fraction of inspired oxygen, 0.6; intermittent mandatory ventilation, 10 breaths/min; tidal volume, 650 mL; positive end-expiratory pressure (PEEP), 15 cm H2O. Arterial blood gases are pH, 7.39; PaCO2, 42 mm Hg; HCO3, 24 mEq/L; and PaO2, 70 mm Hg. The purpose of using PEEP in the treatment of this patient is to: a. increase pulmonary compliance.b. decrease the chance of barotrauma. c. increase alveolar surface tension. d. decrease intrapulmonary shunt.

Correct answer: dRationale: PEEP has three primary purposes: to increase the driving pressure of oxygen, to decrease surface tension and the work of breathing, and to decrease shunt by reopening collapsed alveoli. In ARDS the purpose of PEEP is to open alveoli that have collapsed (called alveolar recruitment) and to keep alveoli open that are still open. The effect of this action is to decrease intrapulmonary shunt.Test-Taking Strategy: Eliminate option b because you know that PEEP increases the risk of barotrauma such as pneumothorax. PEEP decreases surface tension by keeping the alveoli open at the end of expiration, so eliminate option c. PEEP does not increase the compliance of the lung (option a), but it does decrease shunt by opening and keeping open collapsed alveoli.<System> Pulmonary<Section> Mechanical Ventilation; Acute Respiratory Distress Syndrome<Level> Application/Analysis

91. Which of the following assessment findings is specific to hemothorax rather than pneumothorax?a. Chest or shoulder painb. Diminished or absent breath sounds c. Tachypnead. Dullness to percussion

Correct answer: dRationale: Pneumothorax is hyperresonant or even tympanic to percussion, and hemothorax is dull to flat. Options a, b, and c would be seen in both.Test-Taking Strategy: Percussion helps to differentiate various densities such as fluid versus air.<System> Pulmonary<Section> Thoracic Trauma<Level> Knowledge/Comprehension

52. Which of the following is indicated by dyspnea with a normal pulmonary artery occlusive pressure (PAOP), an increase in pulmonary artery diastolic pressure (PAd) and pulmonary vascular resistance, and an increase in right atrial pressure (RAP)?a. Cardiac tamponadeb. Left ventricular failurec. Myocardial infarctiond. Pulmonary embolism

Correct answer: dRationale: Pulmonary embolism causes pulmonary hypertension as evidenced by the elevation of PAd to 5 mm greater than the PAOP and increase in pulmonary vascular resistance (PVR). Cardiac tamponade would cause elevation and equalization of RAP, PAd, and PAOP. Left ventricular failure would elevate PAd and PAOP. Myocardial infarction would not cause any of these changes directly, but if there is left ventricular failure or cardiogenic shock, there would be elevation of PAd and PAOP.Test-Taking Strategy: Increase in PAd with normal PAOP and increase in PVR indicate pulmonary hypertension. The only option that is associated with pulmonary hypertension is pulmonary embolism. Choose option d. In addition, note that options a, b, and c are cardiac and that option d is pulmonary.<System> Pulmonary<Section> Pulmonary Embolism<Level> Synthesis/Evaluation

22. If a patient with acute respiratory failure is acidotic, what would the SaO2 approximately be for a PaO2 of 60?a. 99%b. 92% c. 90% d. 88%

Correct answer: dRationale: The oxyhemoglobin dissociation curve illustrates the relationship between the partial pressure of oxygen in the arterial blood (PaO2) and the saturation of the hemoglobin in the arterial blood (SaO2). For a "normal" curve (e.g., no pH imbalance, normal temperature, normal levels of 2,3-diphosphoglycerate [2, 3-DPG]), the SaO2 is approximately 99% for a PaO2 of 100 mm Hg, 95% for a PaO2 of 80 mm Hg, 90% for a PaO2 of 60 mm Hg, and 75% for a PaO2 of 40 mm Hg. Acidosis, hyperthermia, and increased levels of 2,3-DPG shift the curve to the right. This decreases the affinity between hemoglobin and oxygen, so the SaO2 would be lower than 90% for a PaO2 of 60 mm Hg. This decrease in affinity impairs pickup at the lung level but improves oxygen drop-off at the tissue level.Test-Taking Strategy: Acidosis equals right equals decreased affinity; alkalosis equals left equals increased affinity.<System> Pulmonary<Section> Hypoxia<Level> Synthesis/Evaluation

59. A 52-year-old, 65-kg woman is 2 days after cardiac surgery. She is on a mechanical ventilator. Ventilator Settings Fraction of inspired oxygen (FiO2) 0.6 Tidal volume 600 mL Intermittent mandatory ventilation (IMV) 12 breaths/min Positive end-expiratory pressure (PEEP) 5 cm H2) Arterial Blood Gases pH 7.42 PaCO2 38 mm Hg PaO2 52 mm Hg HCO3 25 mEq/L What ventilator change should be made? a. Increase tidal volume.b. Increase the IMV rate.c. Increase FiO2. d. Increase PEEP.

Correct answer: dRationale: Ventilation is adequate as evidenced by normal PaCO2. Tidal volume and IMV rate do not need to change. PaO2 is too low. This can be improved by increasing the FiO2 or increasing the PEEP. Because the FiO2 is already 0.6 and the PEEP is at only 5 cm (considered a physiologic but not therapeutic level), increasing the PEEP would be preferable. Efforts should be made to limit the amount of time with FiO2 of 1 to no more than 24 hours and with FiO2 of greater than 0.6 to no more than 2 to 3 days to prevent oxygen toxicity.Test-Taking Strategy: Identify the problem. The problem is oxygenation, not ventilation. Oxygenation is improved by increasing PaO2 or increasing PEEP. Because the PEEP is not even at therapeutic levels and FiO2 is at the upper safe limit for more than 2 to 3 days, choose option d.<System> Pulmonary<Section> Mechanical Ventilation<Level> Application/Analysis


Kaugnay na mga set ng pag-aaral

Chapter 3(The Constitution)-Government

View Set

Chapter 2.1: The chemical foundations of life

View Set

chapter 13 spinal cord and spinal nerves

View Set

1.นักวิชาการขนส่งปฏิบัติการ/แนวข้อสอบ พ.ร.บ. ระเบียบข้าราชการพลเรือน พ.ศ. 2551 และที่แก้ไขเพิ่มเติมฉบับที่ 3 พ.ศ. 2562

View Set

Chapter 2 - Understanding the Sky

View Set

Biomechanics of Sport and Exercise Chapter One: Forces

View Set